Nothing Special   »   [go: up one dir, main page]

Ocular Pathology Case Reviews - Azari, Amir (SRG) PDF

Download as pdf or txt
Download as pdf or txt
You are on page 1of 423

Ocular Pathology

Case Reviews
Executive Content Strategist: Russell Gabbedy
Senior Content Development Specialist: Sharon Nash
Senior Project Manager: Beula Christopher
Design: Miles Hitchen
Marketing Manager(s) (UK/USA): Gaynor Jones/Kathleen Reid
Ocular Pathology
Case Reviews
Amir A. Azari, MD
Clinical Assistant Professor
Wills Eye Institute
Jefferson Medical College of Thomas Jefferson University
Philadelphia, PA, USA

Daniel M. Albert, MD, MS


F.A. Davis Professor, Department of Ophthalmology and Visual Sciences, University of
Wisconsin School of Medicine and Public Health
Founding Director, University of Wisconsin McPherson Eye Research Institute
Madison, WI, USA

London,╇ New York,╇ Oxford,╇ Philadelphia,╇ St Louis,╇ Sydney,╇ Toronto╇ 2015


© 2015, Elsevier Inc. All rights reserved.

No part of this publication may be reproduced or transmitted in any form or by any


means, electronic or mechanical, including photocopying, recording, or any
information storage and retrieval system, without permission in writing from the
publisher. Details on how to seek permission, further information about the
Publisher’s permissions policies and our arrangements with organizations such as the
Copyright Clearance Center and the Copyright Licensing Agency, can be found at our
website: www.elsevier.com/permissions.
This book and the individual contributions contained in it are protected under
copyright by the Publisher (other than as may be noted herein).

Notices
Knowledge and best practice in this field are constantly changing. As new research and
experience broaden our understanding, changes in research methods, professional
practices, or medical treatment may become necessary.
Practitioners and researchers must always rely on their own experience and
knowledge in evaluating and using any information, methods, compounds, or
experiments described herein. In using such information or methods they should be
mindful of their own safety and the safety of others, including parties for whom they
have a professional responsibility.
With respect to any drug or pharmaceutical products identified, readers are advised
to check the most current information provided (i) on procedures featured or (ii) by
the manufacturer of each product to be administered, to verify the recommended dose
or formula, the method and duration of administration, and contraindications. It is
the responsibility of practitioners, relying on their own experience and knowledge of
their patients, to make diagnoses, to determine dosages and the best treatment for
each individual patient, and to take all appropriate safety precautions.
To the fullest extent of the law, neither the Publisher nor the authors, contributors,
or editors, assume any liability for any injury and/or damage to persons or property as
a matter of products liability, negligence or otherwise, or from any use or operation of
any methods, products, instructions, or ideas contained in the material herein.
Proudly sourced and uploaded by [StormRG]
ISBN: 978-0-323-28795-1 Kickass Torrents | TPB | ET | h33t

e-book ISBN: 978-0-323-28796-8


Printed in China
Last digit is the print number:╇ 9╅ 8╅ 7╅ 6╅ 5╅ 4╅ 3╅ 2╅ 1

The
publisher’s
policy is to use
paper manufactured
from sustainable forests
Contents

Preface vii
Acknowledgments ix
Dedication xi

CHAPTER 1: EYELID 1

CHAPTER 2: ORBIT AND OPTIC NERVE 83

CHAPTER 3: CORNEA AND CONJUNCTIVA 135

CHAPTER 4: UVEAL TRACT TUMORS 277

CHAPTER 5: RETINA AND VITREOUS 313

CHAPTER 6: GLAUCOMA AND OTHER DISORDERS 359

Index of Cases 401


Index 403

v
This page intentionally left blank
Preface

The goal of this book is to provide clinicians with the ability to look at images of the
important lesions of the eye and adnexa so they can more quickly and accurately make
the diagnosis. This facility is achieved by providing case studies with representative clin-
ical pictures, accompanied by the histopathological appearance of that lesion. As resi-
dents and fellows, we gain experience in “sight recognition” of lesions as viewed directly
on the lids and external surface of the eye or by slit-lamp, ophthalmoscope or an
imaging study. Some lesions are sufficiently distinctive to be diagnosed based on “sight
recognition” without analysis. Often, however, they suggest more than one possibility,
i.e., a differential diagnosis. Analysis of both the clinical evidence and histopathology is
necessary to make a final correct diagnosis.
This book highlights 200 frequently encountered or challenging examples of ocular
and adnexal disorders, together with their associated pathologic appearance. It attempts
to simulate what the clinician experiences when examining the patient in a clinical
setting, by presenting each case initially as an “unknown”, without specifically pointing
out to the reader the diagnostic, clinical, and pathologic features that characterize the
diagnosis. The reader must then analyze, on his or her own, the clinical or differential
diagnosis, and then the confirming pathologic diagnosis. Supporting pages for each
case present the same images accompanied by concise labels indicating the key points
to be noted in order to make the correct diagnosis. This enables the reader to determine
if his or her interpretation and diagnosis is accurate.
In this manner, utilizing self-instruction, Ocular Pathology Case Reviews enables oph-
thalmologists to recognize those features necessary to make correct diagnoses. With
repetition, his or her ability to analyze, interpret, and make that diagnosis is reinforced,
and the clinical appearance and histological features become correlated in a manner
closely approximating that of clinical experience.
Clinicians who have a firm grasp of the clinical–pathologic correlation are better able
to meet the challenges of everyday clinical practice, allowing for a high quality and more
enjoyable experience that ultimately benefits patients. Whether used as an initial learn-
ing tool, or as a method of review, the authors believe Ocular Pathology Case Reviews will
be an effective and enjoyable support to you in your practice of ophthalmology.

Amir A. Azari, MD
Daniel M. Albert, MD, MS

vii
This page intentionally left blank
Acknowledgments

The authors express their gratitude to Drs Mozghan Rezai Kanavi and Heather Potter for
their assistance and encouragement in compiling this book. Drs Neal Barney, Sarah
Nehls, Jason Sokol, and Justin Gottlieb generously helped in providing clinical
photographs. Laura Cruz and Patty Rasmussen provided valuable assistance in its
preparation. Vicky Rogness has meticulously prepared the high quality slides that are
used in this book. Russell Gabbedy, Executive Content Strategist, and Sharon Nash,
Senior Content Development Specialist, of Elsevier guided this book through the
editorial and publishing process with expertise, insight, and graciousness.

ix
This page intentionally left blank
Dedication

This book is dedicated to


our wives for their love, patience, and support

xi
This page intentionally left blank
Chapter 1â•…Eyelid CASE 1 â•…

A 67-year-old woman presented with a left lower eyelid lesion. Excisional biopsy of the lesion
demonstrates the histological findings shown below.

1
ANSWER â•… Case 1

Diagnosis:╇ Basal cell carcinoma.

Clinical description:╇ A raised telangiectatic lesion with associated madarosis is present in the left lower
eyelid.

Histological description:╇ Histopathology demonstrates tissue covered with keratinized stratified squamous
epithelium. The dermis contains cords, and islands of basaloid cells, which exhibit peripheral palisading of
their nuclei. A retraction artifact is seen.

Islands of basaloid cells

Peripheral
palisading of nuclei
Retraction
artifact

2
Chapter 1â•…Eyelid CASE 2 â•…

A 68-year-old woman presented with a red, itchy left lower eyelid lesion present for the past 3
months.

3
ANSWER â•… Case 2

Diagnosis:╇ Actinic keratosis.

Clinical description:╇ A red, minimally elevated, scaly lesion at the margin of the left lower lid is
demonstrated.

Histological description:╇ Variable amounts of hyperkeratosis (increased thickness of the keratin layer),
parakeratosis (presence of nuclei in the keratin layer), and acanthosis (thickening of the prickle cell layer) is
seen. There is some atypia in the basal layer. Mild nongranulomatous inflammation is seen within the dermis.

Hyperkeratosis

Parakeratosis: presence
of many nuclei within
Nongranulomatous the keratin layer
inflammation

Nongranulomatous
inflammation
Solar elastosis

4
Chapter 1â•…Eyelid CASE 3 â•…

A 67-year-old woman presented with a 5-month history of a right eyelid lesion.

5
ANSWER â•… Case 3

Diagnosis:╇ Squamous cell papilloma.

Clinical description:╇ Clinical examination reveals a partially keratinized papillomatous right upper eyelid
lesion.

Histological description:╇ Histopathology reveals a lesion covered by keratinized stratified squamous


epithelium and exhibiting acanthosis and papillary projections.

Keratin layer

Acanthotic skin
epithelium

Dermis

Pilar units

6
Chapter 1â•…Eyelid CASE 4 â•…

A 72-year-old woman presented with multiple eyelid lesions.

7
ANSWER â•… Case 4

Diagnosis:╇ Verruca vulgaris.

Clinical description:╇ Clinical examination shows multiple keratinized lesions on the upper and lower
eyelids.

Histological description:╇ Histopathology reveals fronds of hyperkeratotic stratified squamous epithelium


lining a fibrovascular dermis.

Keratin

Keratin

Keratin

Fibrovascular core

8
Chapter 1â•…Eyelid CASE 5 â•…

A 61-year-old man presented with multiple skin tags. The skin tags were removed in the doctor’s
office.

9
ANSWER â•… Case 5

Diagnosis:╇ Seborrheic keratosis (sessile).

Clinical description:╇ Multiple pigmented lesions with a “stuck-on” appearance are seen.

Histological description:╇ Histopathology reveals mildly acanthotic, keratinized, stratified squamous


epithelium with pseudohorn cysts (asterisks).

Surface keratin

Acanthotic, keratinized
*
* squamous epithelium
*
Dermis
Mild chronic nongranulomatous
inflammation

10
Chapter 1â•…Eyelid CASE 6 â•…

A 47-year-old woman presented with a left upper eyelid lesion. Excisional biopsy of the lesion showed
the following histological findings.

11
ANSWER â•… Case 6

Diagnosis:╇ Xanthelasma.

Clinical description:╇ Clinical examination shows a yellow, elevated lesion on the left upper eyelid.

Histological description:╇ Histopathology reveals a collection of lipid-laden histiocytes within the dermis.

Skin epithelium

Foamy lipid-laden histiocytes


within the dermis

12
Chapter 1â•…Eyelid CASE 7 â•…

From Albert, Daniel M., Miller, Joan W., Azar, Dimitri T., and Blodi,
Barbara A. (eds). 2008. Albert & Jakobiec’s Principles and Practice of
Ophthalmology, 3rd ed. Philadelphia: Copyright Elsevier 2008.

A 62-year-old man presented with a left lower eyelid lesion.

13
ANSWER â•… Case 7

Diagnosis:╇ Sebaceous cell adenoma.

Clinical description:╇ Clinical examination demonstrates a large cystic left lower eyelid lesion.

Histological description:╇ Histopathology demonstrates skin with keratinized squamous epithelium. In the
dermis, there are hyperplastic sebaceous glands forming multiple cavities containing proteinaceous material
(asterisks). Sebaceous adenoma of the skin may be associated with Muir–Torre syndrome.

Surface epithelium

*
*
* * *
*
*
*
*

14
Chapter 1â•…Eyelid CASE 8 â•…

The patient is a 48-year-old man with a 3-month history of a left lower eyelid lesion. The clinical
appearance is depicted above.

15
ANSWER â•… Case 8

Diagnosis:╇ Inverted follicular keratosis.

Clinical description:╇ External examination reveals a keratinized left lower eyelid mass.

Histological description:╇ The excised lesion demonstrates hyperkeratosis, parakeratosis, and acanthosis. The
tissue has an overall inverted papillary pattern. Squamous eddies are seen.

Hyperkeratosis and
parakeratosis

Acanthosis

Squamous eddies

16
Chapter 1â•…Eyelid CASE 9 â•…

A 74-year-old woman presented with the skin changes seen above.

17
ANSWER â•… Case 9

Diagnosis:╇ Rosacea.

Clinical description:╇ Facial and periocular erythema is noted.

Histological description:╇ Histopathology reveals keratinized, stratified squamous epithelium with


acanthosis and focal parakeratosis. Granulomatous inflammation surrounding some of the hair follicles is
present within the dermis.

Hair follicle Granulomatous


inflammation

18
Chapter 1â•…Eyelid CASE 10 â•…

A 57-year-old man presented with multiple pigmented lesions.

19
ANSWER â•… Case 10

Diagnosis:╇ Seborrheic keratosis (papillomatous).

Clinical description:╇ Multiple brown skin tags in the left lower lid can be seen.

Histological description:╇ Histopathology reveals tissue lined by keratinized stratified squamous epithelium
with a few pilosebaceous units and focal basal epithelial pigmentation. There is hyperkeratosis and
papillomatous acanthosis. Scattered pseudohorn cysts (asterisks) are also present. Focal areas of chronic
inflammation are noted in the dermis.

*
*
Basal epithelial
pigmentation

Focal areas of
inflammation

20
Chapter 1â•…Eyelid CASE 11 â•…

A 53-year-old man presented with a 3-month history of a left lower eyelid lesion.

21
ANSWER â•… Case 11

Diagnosis:╇ Keratoacanthoma.

Clinical description:╇ Histopathology reveals a skin lesion with central crater filled with keratin. The edges of
the lesion are rolled-up and elevated.

Histological description:╇ Histopathological evaluation reveals a keratinized stratified squamous surface


epithelium with a crater-shaped endophytic lesion filled with keratin. Acanthosis and parakeratosis are present.
Mild atypia and a few mitotic figures are noted.

Normal skin
epithelium

Central mass of Central mass of


keratin keratin

Acanthosis

Acanthosis

Normal skin
epithelium

22
Chapter 1â•…Eyelid CASE 12 â•…

From Albert, Daniel M., Miller, Joan W., Azar, Dimitri T., and Blodi, Barbara A. (eds). 2008. Albert
& Jakobiec’s Principles and Practice of Ophthalmology, 3rd ed. Philadelphia: Copyright Elsevier 2008.

An 81-year-old man presented with an enlarging lesion on his left brow.

23
ANSWER â•… Case 12

Diagnosis:╇ Squamous cell carcinoma in situ.

Clinical description:╇ Clinical examination reveals an erythematous lesion with scaling.

Histological description:╇ Sections through the specimen show a thickened, keratinized, stratified squamous
epithelium with marked hyperkeratosis and parakeratosis. Nests of malignant squamous cells are present
(asterisks). Intraepithelial keratin pearls are also seen (short arrow).

Skin epithelium

Acanthosis

*
*

24
Chapter 1â•…Eyelid CASE 13 â•…

From Albert, Daniel M., Miller, Joan W., Azar, Dimitri T., and Blodi, Barbara A. (eds). 2008. Albert
& Jakobiec’s Principles and Practice of Ophthalmology, 3rd ed. Philadelphia: Copyright Elsevier.
2008.

A 67-year-old man presented with a 3-month history of an eyelid lesion.

25
ANSWER â•… Case 13

Diagnosis:╇ Invasive squamous cell carcinoma.

Clinical description:╇ Clinical examination reveals an ulcerated and scarred right upper eyelid lesion.

Histological description:╇ Sections through the specimen show keratinized, stratified squamous epithelium.
Nests of malignant squamous cells infiltrate the dermis (asterisks). Keratin pearls are also present (arrows).

Surface epithelium

* *
*

* *

26
Chapter 1â•…Eyelid CASE 14 â•…

A 69-year-old woman presented with an ulcerated skin lesion. She underwent excisional biopsy.

27
ANSWER â•… Case 14

Diagnosis:╇ Morpheaform basal cell carcinoma.

Clinical description:╇ Clinical examination reveals an ulcerated left lower eyelid lesion.

Histological description:╇ Histopathology demonstrates keratinized stratified squamous epithelium overlying


a dermis infiltrated with bands and cords of flattened tumor cells.

Keratinized
surface
epithelium

Hair follicle

Cords of tumor cells

Sclerosed
dermis

28
Chapter 1â•…Eyelid CASE 15 â•…

Courtesy of Shahed Ghoghawala M.D. University of Wisconsin-Madison.

A 66-year-old man presented with a right lower eyelid lesion. The lesion had been present for over a
year and was increasing in size.

29
ANSWER â•… Case 15

Diagnosis:╇ Sebaceous cell carcinoma with pagetoid spread.

Clinical description:╇ Clinical examination reveals cystic lesions in the right lower eyelid with an associated
madarosis.

Histological description:╇ Histopathology reveals tissue lined by keratinized stratified squamous epithelium.
Cords and lobules of foamy cells lined by basaloid cells are present within the dermis and subepithelium and
infiltrate into the epithelium (pagetoid spread). Higher magnification (bottom) demonstrates the foamy cells
in more detail.

Early pagetoid
spread
Foamy cells lined by
basaloid cells

30
Chapter 1â•…Eyelid CASE 16 â•…

A 65-year-old man presented with several pigmented nodules present on his upper and lower left
eyelids.

31
ANSWER â•… Case 16

Diagnosis:╇ Pigmented basal cell carcinoma.

Clinical description:╇ Ulcerated left lower eyelid lesion surrounded by a few pigmented nodular lesions is
seen. A few pigmented lesions are also present in the left upper eyelid.

Histological description:╇ Histopathology reveals tissue lined by keratinized stratified squamous epithelium.
There are extensive islands of atypical basaloid cells with high nuclear to cytoplasmic ratio in the dermis. The
islands demonstrate peripheral palisading and retraction artifact (arrow). Dense pigmentation is present.
Differentiation from melanoma was confirmed on bleach sections (not shown).

32
Chapter 1â•…Eyelid CASE 17 â•…

From Albert, Daniel M., Miller, Joan W., Azar, Dimitri T., and Blodi, Barbara A. (eds). 2008. Albert
& Jakobiec’s Principles and Practice of Ophthalmology, 3rd ed. Philadelphia. Copyright Elsevier 2008.

An 81-year-old man presented with eyelid lesions.

33
ANSWER â•… Case 17

Diagnosis:╇ Merkel cell carcinoma.

Clinical description:╇ Slit-lamp examination demonstrates two elevated eyelid nodules in the right lower lid.

Histological description:╇ Histopathological examination reveals a skin with keratinized stratified squamous
epithelium. In the dermis, there are sheets of atypical and pleomorphic cells with finely dispersed chromatin
and inconspicuous nucleoli.

Keratinized skin
epithelium

Dermis infiltrated with atypical and pleomorphic cells

34
Chapter 1â•…Eyelid CASE 18 â•…

A 72-year-old man presented with a 6-month history of a growing eyelid mass.

35
ANSWER â•… Case 18

Diagnosis:╇ Eccrine hidrocystoma.

Clinical description:╇ Clinical examination shows a large, translucent fluid-filled eyelid cyst.

Histological description:╇ Multilocular branching lumen (asterisks) filled with eosinophilic material is seen
on histopathology. The lumen is lined by a double layer of epithelium. No apical snouts are seen.

*
*
Lumen filled with
eosinophilic material

Dual epithelial
lining

36
Chapter 1â•…Eyelid CASE 19 â•…

A 53-year-old man presented with multiple cystic lesions in the medial canthus and upper eyelid. He
underwent excisional biopsy of the lesions.

37
ANSWER â•… Case 19

Diagnosis:╇ Apocrine hidrocystoma.

Clinical description:╇ Multiple fluid filled, translucent vesicles in the medial canthus and upper eyelid.

Histological description:╇ Large intradermal cystic cavity lined by a double epithelial layer. The innermost
cells display apical snouts (decapitation bodies).

Surface
epithelium of skin

Intradermal
cystic cavity
Epithelial lining

Dual epithelial Apical snouts


lining

38
Chapter 1â•…Eyelid CASE 20 â•…

A 58-year-old man presented with a slow-growing right lower eyelid lesion. He later underwent an
excisional biopsy of the lesion.

39
ANSWER â•… Case 20

Diagnosis:╇ Epithelial inclusion cyst.

Clinical description:╇ Raised white/opaque lesion is seen in the right lower eyelid margin.

Histological description:╇ Large cystic space lined by keratinized stratified squamous epithelium is seen
within the dermis. Unlike dermoid cysts, the epithelial lining does not have pilosebaceous units.

Surface epithelium of the


overlying skin

Cystic space

Keratinized stratified squamous


epithelial lining of the cyst

Cystic space

40
Chapter 1â•…Eyelid CASE 21 â•…

The patient is a 21-year-old woman who presented with multiple skin lesions under the right lower
eyelid.

41
ANSWER â•… Case 21

Diagnosis:╇ Syringoma.

Clinical description:╇ Multiple flesh-colored skin lesions are seen.

Histological description:╇ Histopathology reveals keratinized stratified squamous epithelium overlying


numerous single and double-layered epithelial strands with cysts (asterisks) and some with “tadpole
configuration” (arrowheads) present throughout the dermis.

*
*

42
Chapter 1â•…Eyelid CASE 22 â•…

A 35-year-old woman presented with a 3-month history of an eyelid lesion.

43
ANSWER â•… Case 22

Diagnosis:╇ Infundibular cyst.

Clinical description:╇ Clinical examination reveals a subcutaneous mass in the right upper eyelid.

Histological description:╇ The slides reveal tissue lined by a keratinized stratified squamous epithelium. A
follicular infundibular structure, filled with keratin, is seen within the dermis (circle).

Mild inflammation Keratin Surface epithelium

Dermis

44
Chapter 1â•…Eyelid CASE 23 â•…

An 84-year-old female presented with a 6-month history of a left upper eyelid lesion.

45
ANSWER â•… Case 23

Diagnosis:╇ Pilomatrixoma.

Clinical description:╇ A raised, nodular lesion, with a few dilated vessels, is seen in the left upper eyelid.

Histological description:╇ Histopathology demonstrates islands of shadow cells and basophilic cells
intermixed with numerous epithelioid histiocytes. Calcium is present.

Calcification

Epithelioid histiocytes

Shadow cells

Basophilic cells

46
Chapter 1â•…Eyelid CASE 24 â•…

A 76-year-old woman presented with a rapidly growing left lower eyelid lesion.

47
ANSWER â•… Case 24

Diagnosis:╇ Trichoepithelioma.

Clinical description:╇ A raised pearly lesion with a smooth surface is noted.

Histological description:╇ The slides reveal stratified squamous epithelium overlying a dermis with multiple
small nests of basaloid cells with peripheral palisading of their nuclei (asterisks).

Surface epithelium

* *

48
Chapter 1â•…Eyelid CASE 25 â•…

From Albert, Daniel M., Miller, Joan W., Azar, Dimitri T., and Blodi, Barbara A. (eds). 2008. Albert
& Jakobiec’s Principles and Practice of Ophthalmology, 3rd ed. Philadelphia: Copyright Elsevier 2008.

A 35-year-old woman presented with a 3-month history of an eyelid lesion.

49
ANSWER â•… Case 25

Diagnosis:╇ Trichofolliculoma.

Clinical description:╇ Clinical examination reveals a cystic lesion in the right upper eyelid with a wisp of
lanugo hairs emanating from it.

Histological description:╇ The slides reveal tissue lined by a keratinized stratified squamous epithelium. A
follicular infundibular structure, surrounded by multiple small follicles (asterisks), containing keratin, and
communicating with the skin surface can be seen.

Area of communication
with surface *
*

50
Chapter 1â•…Eyelid CASE 26 â•…

A 43-year-old woman presented with a nodular right lower eyelid lesion.

51
ANSWER â•… Case 26

Diagnosis:╇ Trichilemmoma.

Clinical description:╇ Clinical examination reveals a round, nodular, flesh-colored right lower eyelid skin
lesion.

Histological description:╇ The histopathology reveals hyperkeratotic stratified squamous epithelium with
marked parakeratosis and acanthosis. Areas of well-defined and plate-like clear cells (asterisk) and epithelial
down-growth into the dermis are seen. Areas of palisading are also seen.

Acanthosis

Peripheral palisading

52
Chapter 1â•…Eyelid CASE 27 â•…

An 18-year-old man presented with a 1-week history of a right upper eyelid nodule refractory to
treatment with warm compress and eyelid massage.

53
ANSWER â•… Case 27

Diagnosis:╇ Chalazion.

Clinical description:╇ A red, elevated right upper eyelid mass is seen.

Histological description:╇ Granulomatous and nongranulomatous chronic inflammation is seen surrounding


a large lipid vacuole. Multinucleated giant cells are also seen.

Nongranulomatous
chronic inflammation

Lipid
vacuole

Multinucleated giant cell


Granulomatous
inflammation

54
Chapter 1â•…Eyelid CASE 28 â•…

A 38-year-old woman presented with multiple eyelid skin lesions.

55
ANSWER â•… Case 28

Diagnosis:╇ Molluscum contagiosum.

Clinical description:╇ Clinical examination reveals multiple pearly flesh-colored dome-shaped lesions.

Histological description:╇ Histopathology reveals a discrete dome-shaped lesion with an umbilicated center.
Pear-shaped acanthotic lobules of epithelial cells filled with eosinophilic intracytoplasmic inclusion bodies
(Henderson–Patterson corpuscles) are seen. The inclusion bodies (circles) are deeply eosinophilic in the lower
cells and become less eosinophilic towards the level of the granular layer.

Acanthotic lobules of
epithelial cells

56
Chapter 1â•…Eyelid CASE 29 â•…

A 58-year-old woman presented with a right lower eyelid lesion. Excisional biopsy of the lesion shows
the following histological findings.

57
ANSWER â•… Case 29

Diagnosis:╇ Intradermal nevus.

Clinical description:╇ Clinical examination reveals a pearly, flesh-colored, well-circumscribed raised lesion.

Histological description:╇ Histopathology reveals nests of benign melanocytic cells confined to the dermis. A
cell free region known as the “grenz zone” is seen beneath the epithelium.

Epithelium

Grenz zone

Nests of nevus cells within the dermis

58
Chapter 1â•…Eyelid CASE 30 â•…

A 33-year-old woman presented with an eyelid lesion. The lesion was removed and evaluated
histopathologically.

59
ANSWER â•… Case 30

Diagnosis:╇ Compound nevus.

Clinical description:╇ A mostly flat, darkly pigmented lesion at the margin of the left lower eyelid is noted.

Histological description:╇ Nests and nodules of ovoid basaloid cells are present within the dermis as well as
at the junction between the epidermis and the dermis.

Keratinized stratified
squamous epithelium
of the skin (epidermis)

Junctional component
Dermal component of of the nevus
the nevus

Pilosebaceous units

60
Chapter 1â•…Eyelid CASE 31 â•…

A 62-year-old woman presented with a pigmented lesion in the right lower eyelid.

61
ANSWER â•… Case 31

Diagnosis:╇ Malignant melanoma of the eyelid.

Clinical description:╇ Clinical examination reveals a slightly raised pigmented skin lesion.

Histological description:╇ Histopathology demonstrates skin infiltrated with pigmented atypical melanocytes
(arrows) consistent with melanoma.

62
Chapter 1â•…Eyelid CASE 32 â•…

Courtesy of Neal P. Barney, M.D. University of Wisconsin-Madison

A 44-year-old obese male presented with dry, irritated eyes.

63
ANSWER â•… Case 32

Diagnosis:╇ Floppy eyelid syndrome.

Clinical description:╇ Upper eyelid laxity is apparent. The tarsal conjunctiva has a velvety appearance.

Histological description:╇ Biopsy shows multiple conjunctival papillae marked by elevated nodules
infiltrated with inflammatory cells (arrows) and containing fronds of vascular core (arrowheads).

Meibomian glands
within the tarsus

64
Chapter 1â•…Eyelid CASE 33 â•…

A 63-year-old man presented with a chief complaint of droopy eyelids. He underwent a functional
blepharoplasty.

65
ANSWER â•… Case 33

Diagnosis:╇ Dermatochalasis.

Clinical description:╇ Bilateral ptosis and redundancy of the upper eyelid skin is noted on clinical
examination.

Histological description:╇ Histopathology reveals keratinized stratified squamous epithelium with associated
pilosebaceous units overlying a dermis with loosely arranged collagen fibers.

Keratinized stratified
squamous epithelium Sebaceous gland
Pilar unit

Loosely arranged collagen fibers

66
Chapter 1â•…Eyelid CASE 34 â•…

A 3-year-old child was brought in by his mother for evaluation of abnormal eyelids.

67
ANSWER â•… Case 34

Diagnosis:╇ Entropion with normal histology.

Clinical description:╇ Clinical examination demonstrates bilateral lower eyelid entropion.

Histological description:╇ The slides reveal tissue lined by keratinized stratified squamous epithelium with
associated pilosebaceous units, muscle, and fat (normal skin).

Keratinized squamous
epithelium

Pilar unit

Dermis

Striated muscles

68
Chapter 1â•…Eyelid CASE 35 â•…

A 44-year-old man presented with an eyelid lesion. The lesion had been present for many years
causing severe ptosis.

69
ANSWER â•… Case 35

Diagnosis:╇ Cavernous hemangioma of the eyelid.

Clinical description:╇ Clinical examination shows a corrugated, reddish-purple elevated right upper eyelid
lesion.

Histological description:╇ Histopathology shows large, dilated vascular channels that are lined by a single
layer of flat endothelium. The vascular channels are filled with red blood cells.

Dilated vascular channels

Intervening septae
Endothelial lining

70
Chapter 1â•…Eyelid CASE 36 â•…

The patient is a 58-year-old woman who presented with a right upper eyelid lesion.

71
ANSWER â•… Case 36

Diagnosis:╇ Eyelid varix.

Clinical description:╇ Clinical examination reveals a vascular cutaneous lesion on the right upper eyelid.

Histological description:╇ Histopathology demonstrates a few thin-walled dilated vascular channels lined by
a monolayer of endothelial cells and filled with red blood cells.

Lining of the vascular


channel

72
Chapter 1â•…Eyelid CASE 37 â•…

A 2-year-old infant has had the eyelid finding shown above since birth. He later underwent an
excisional biopsy of the right upper eyelid lesion.

73
ANSWER â•… Case 37

Diagnosis:╇ Eyelid capillary hemangioma.

Clinical description:╇ An elevated reddish right upper eyelid swelling is seen.

Histological description:╇ Proliferation of lobulated capillaries, lined by a single layer of endothelial cells is
seen within the dermis. The capillaries are separated by fibrous septae.

Proliferating capillaries

Intervening fibrous septa

74
Chapter 1â•…Eyelid CASE 38 â•…

A 48-year-old man presented with redness and swelling of his right upper eyelid. He underwent
excisional biopsy of the eyelid.

75
ANSWER â•… Case 38

Diagnosis:╇ Melkersson–Rosenthal syndrome.

Clinical description:╇ Right upper eyelid edema and erythema is noted.

Histological description:╇ Histopathology demonstrates skin tissue lined by keratinized stratified squamous
epithelium. Granulomatous inflammation is seen within the blood vessel lumen, a finding that is consistent
with Melkersson–Rosenthal syndrome.

Keratinized stratified
squamous epithelium

Blood vessel lumen

Intravascular granulomatous
inflammation

76
Chapter 1â•…Eyelid CASE 39 â•…

A 2-year-old child presented with yellowish skin lesions present since birth.

77
ANSWER â•… Case 39

Diagnosis:╇ Nevus sebaceous.

Clinical description:╇ Clinical examination reveals multiple elevated and flat skin lesions above the medial
canthus.

Histological description:╇ Histopathology reveals tissue lined by a keratinized stratified squamous


epithelium with associated hair follicles and numerous sebaceous glands. The lesion is hyperkeratotic,
acanthotic, and papillomatous with focal areas of parakeratosis.

Keratinized skin epithelium

Sebaceous glands
Sebaceous glands

78
Chapter 1â•…Eyelid CASE 40 â•…

A 64-year-old woman presented with multiple lesions on her face.

79
ANSWER â•… Case 40

Diagnosis:╇ Face and eyelid cavernous hemangioma.

Clinical description:╇ Clinical examination reveals multiple flat and elevated vascular and pigmented skin
lesions.

Histological description:╇ Histopathological evaluation reveals lobules of blood-filled dilated vascular


channels lined by a flat endothelial layer.

Dilated blood-filled
vascular channels

Endothelial lining

80
Chapter 1â•…Eyelid CASE 41 â•…

A 56-year-old man presented with a growing eyelid mass. He later underwent excisional biopsy of the
eyelid lesions.

81
ANSWER â•… Case 41

Diagnosis:╇ Angiolymphoid hyperplasia with eosinophilia (ALHE).

Clinical description:╇ Clinical examination reveals round, flesh-colored growths over the upper and lower
eyelids.

Histological description:╇ Histopathology reveals vascular proliferation (asterisks) with vessels


demonstrating intracytoplasmic vacuolization of their endothelium (arrowheads). Numerous eosinophils,
plasma cells, and lymphocytes are also seen.

Eosinophils
Plasma cells

*
*
*

82
Chapter 2â•… Orbit and optic nerve CASE 42 â•…

A 45-year-old woman presented with left upper eyelid fullness. Excisional biopsy of the lesion shows
the histological findings shown below.

83
ANSWER â•… Case 42

Diagnosis:╇ Pleomorphic adenoma (benign mixed tumor).

Clinical description:╇ Mild fullness of the left upper eyelid is noted.

Histological description:╇ The tumor contains ductules and mesenchymal elements. The ductules are
composed of dual epithelial layers. There is differentiation of the outer epithelial layer into mesenchymal
elements such as fat, cartilage, and bone.

Mesenchymal matrix

Dual layer
Myoepithelial cells ducts

84
Chapter 2â•… Orbit and optic nerve CASE 43 â•…

From Albert, Daniel M., Miller, Joan W., Azar, Dimitri T., and Blodi, Barbara A. (eds). 2008. Albert
& Jakobiec’s Principles and Practice of Ophthalmology, 3rd ed. Philadelphia: Copyright Elsevier 2008.

A 34-year-old woman presented with pain and swelling of her left upper eyelid for the past week.
The patient’s CT scan is shown above.

85
ANSWER â•… Case 43

Diagnosis:╇ Dacryoadenitis.

Clinical description:╇ A CT scan of the orbit shows enlargement of the lacrimal gland consistent with
dacryoadenitis.

Histological description:╇ Histopathology reveals lacrimal gland tissue that contains aggregates of chronic
nongranulomatous, lymphocytic inflammation without necrosis (circles). CD3 and CD20 staining do not
show monoclonality (not shown).

Lacrimal gland acini

86
Chapter 2â•… Orbit and optic nerve CASE 44 â•…

From Albert, Daniel M., Miller, Joan W., Azar, Dimitri T., and Blodi, Barbara A. (eds). 2008. Albert
& Jakobiec’s Principles and Practice of Ophthalmology, 3rd ed. Philadelphia: Copyright Elsevier 2008.

A 68-year-old patient presented with pain, diplopia, and proptosis. The patient’s MRI is shown above.

87
ANSWER â•… Case 44

Diagnosis:╇ Adenoid cystic carcinoma.

Clinical description:╇ Magnetic resonance imaging shows a left orbital tumor with erosion of the left lateral
orbital wall.

Histological description:╇ Histopathology demonstrates tumor with classic “Swiss cheese” pattern, consisting
of cysts (asterisks) surrounded by basaloid cells (arrows). The tumor demonstrates perineural invasion.

*
Nerve bundle

88
Chapter 2â•… Orbit and optic nerve CASE 45 â•…

A 59-year-old man presented with double vision and left eye proptosis.

89
ANSWER â•… Case 45

Diagnosis:╇ Hemangiopericytoma.

Clinical description:╇ A well-circumscribed intraconal mass is seen on MRI.

Histological description:╇ The histopathology reveals dilated vascular channels with a “staghorn”
configuration within a cellular matrix composed of proliferated pericytes.

Cellular background

Dilated vascular channel


with “staghorn”
configuration

90
Chapter 2â•… Orbit and optic nerve CASE 46 â•…

A 21-year-old man presented with increasing proptosis over the past 6 months. The photograph above
was taken during an orbital biopsy.

91
ANSWER â•… Case 46

Diagnosis:╇ Cavernous hemangioma of the orbit.

Clinical description:╇ The intraoperative photograph demonstrates a well-circumscribed, vascular orbital


lesion.

Histological description:╇ Histopathology demonstrates well-circumscribed lesions composed of dilated


thick-walled blood vessels lined by endothelium. The vascular spaces are filled with blood.

Dilated blood-filled
vascular channels

92
Chapter 2â•… Orbit and optic nerve CASE 47 â•…

A 64-year-old man recently developed the mass evident in his right eye.

93
ANSWER â•… Case 47

Diagnosis:╇ Prolapsed orbital fat.

Clinical description:╇ Clinical examination reveals prolapsed orbital fat.

Histological description:╇ Histopathology demonstrates lobules of adipose tissue with intervening


fibrovascular septae consistent with prolapsed orbital fat.

94
Chapter 2â•… Orbit and optic nerve CASE 48 â•…

Courtesy of Jason A. Sokol, MD. Director of Oculofacial Plastic and Orbital Surgery at Kansas
University Eye Center.

A 55-year-old man presented with diplopia for the past 6 months. The patient’s CT scan is shown
above.

95
ANSWER â•… Case 48

Diagnosis:╇ Solitary fibrous tumor of the orbit.

Clinical description:╇ T1-weighted magnetic resonance imaging demonstrates an enhancing, well-


circumscribed lesion located superonasally in the right orbit.

Histological description:╇ Sections through the specimen contain a very cellular connective tissue with
spindle-shaped fibrocytes in a storiform pattern. Numerous fine vessels are seen coursing throughout the
specimen. Larger slit-like vessels are also present (asterisks).

96
Chapter 2â•… Orbit and optic nerve CASE 49 â•…

Courtesy of Jason A. Sokol, M.D. Director of Oculofacial Plastic and Orbital Surgery at Kansas
University Eye Center.

A 10-year-old boy presented with a 2-month history of diplopia and exophthalmos of the right eye.

97
ANSWER â•… Case 49

Diagnosis:╇ Rhabdomyosarcoma.

Clinical description:╇ Clinical examination reveals restriction of motility, mild proptosis, and right upper and
lower eyelid fullness.

Histological description:╇ Histopathological review of specimens using Masson trichrome reveals tissue
composed of pleomorphic spindle-shaped cells with large, round dark nuclei. There are noted to be several
areas where there are strap cells with markedly eosinophilic cytoplasm, the configuration of fibrils that reveal
cross striations compatible with primitive skeletal muscle.

Primitive skeletal
muscle

98
Chapter 2â•… Orbit and optic nerve CASE 50 â•…

A 53-year-old patient presented with proptosis and double vision.

99
ANSWER â•… Case 50

Diagnosis:╇ Schwannoma.

Clinical description:╇ T1-weighted magnetic resonance imaging shows a well-circumscribed intraconal mass
in the left orbit.

Histological description:╇ Section through the specimen shows scattered spindle-shaped cells growing in
whorl pattern. There are areas of loosely and randomly spaced collagen bundles. S-100 stain is positive
(not shown).

100
Chapter 2â•… Orbit and optic nerve CASE 51 â•…

An 8-year-old boy presented with the above eyelid deformity.

101
ANSWER â•… Case 51

Diagnosis:╇ Neurofibroma in a patient with neurofibromatosis.

Clinical description:╇ The clinical examination reveals an “S-shaped” eyelid deformity.

Histological description:╇ Histopathological evaluation shows multiple enlarged nerve bundles intertwined
with collagen consistent with neurofibroma (asterisks).

*
* *
*
* *

*
*

102
Chapter 2â•… Orbit and optic nerve CASE 52 â•…

A 23-year-old man presented with diplopia and facial pain.

103
ANSWER â•… Case 52

Diagnosis:╇ Fibrous dysplasia.

Clinical description:╇ Facial asymmetry with swelling of left side of the face is noted.

Histological description:╇ Histopathological examination reveals irregular trabeculae of immature woven


bone (resembling Chinese letters) in a fibrous background.

Woven bone shaped like


Chinese letters

Fibrous background

104
Chapter 2â•… Orbit and optic nerve CASE 53 â•…

From Albert, Daniel M., Miller, Joan W., Azar, Dimitri T., and Blodi, Barbara A. (eds). 2008. Albert
& Jakobiec’s Principles and Practice of Ophthalmology, 3rd ed. Philadelphia: Copyright Elsevier 2008.

A 45-year-old patient presents with ocular pain and diplopia. The patient’s CT scan is shown above.

105
ANSWER â•… Case 53

Diagnosis:╇ Eosinophilic granuloma.

Clinical description:╇ The CT scan demonstrates a chronic lytic superior orbital lesion.

Histological description:╇ Histopathology reveals two populations of cells. One population of cells is
eosinophils (arrows) and the other population is epithelioid giant cells.

106
Chapter 2â•… Orbit and optic nerve CASE 54 â•…

From Albert, Daniel M., Miller, Joan W., Azar, Dimitri T., and Blodi,
Barbara A. (eds). 2008. Albert & Jakobiec’s Principles and Practice of
Ophthalmology, 3rd ed. Philadelphia: Copyright Elsevier 2008.

A 49-year-old female presented with eye pain and restricted ocular motility.

107
ANSWER â•… Case 54

Diagnosis:╇ Wegener’s granulomatosis.

Clinical description:╇ CT scan demonstrates a left orbital mass with erosion through the medial orbital wall.
Patient tested positive for cANCA.

Histological description:╇ Histopathology reveals areas of necrosis surrounded by epithelioid cells as well as
lymphocytes, plasma cells, and a few neutrophils (asterisks).

*
*

*
Necrosis

*
* *

108
Chapter 2â•… Orbit and optic nerve CASE 55 â•…

From Albert, Daniel M., Miller, Joan W., Azar, Dimitri T., and Blodi,
Barbara A. (eds). 2008. Albert & Jakobiec’s Principles and Practice of
Ophthalmology, 3rd ed. Philadelphia: Copyright Elsevier 2008.

A 1-year-old child presented with ptosis and a right superior orbital mass, depicted above.
He underwent an orbital biopsy.

109
ANSWER â•… Case 55

Diagnosis:╇ Juvenile xanthogranuloma with primary orbital involvement.

Clinical description:╇ Clinical examination reveals a right upper eyelid swelling causing ptosis in this
1-year-old patient.

Histological description:╇ Orbital biopsy reveals a histiocytic cell tumor containing numerous Touton
multinucleated giant cells (arrows).

110
Chapter 2â•… Orbit and optic nerve CASE 56 â•…

A 19-year-old woman presented with a 4-month history of left upper eyelid fullness. She underwent
anterior orbitotomy.

111
ANSWER â•… Case 56

Diagnosis:╇ Dermoid cyst.

Clinical description:╇ Left upper eyelid swelling and fullness is noted temporally.

Histological description:╇ A large cystic cavity filled with keratin is seen (top). Higher magnification
(bottom) shows that the cystic cavity is lined by keratinized stratified squamous epithelium and associated
pilosebaceous units.

Lining of the cyst with


associated pilosebaceous
units

Keratin-filled cystic space

Keratinized
stratified squamous
epithelium

Pilar unit with hair


shaft inside

112
Chapter 2â•… Orbit and optic nerve CASE 57 â•…

Courtesy of Jason A. Sokol, MD. Director of Oculofacial Plastic and Orbital Surgery at Kansas
University Eye Center.

A 36-year-old woman presented with pain, redness, and swelling of her right upper eyelid.
The material in the upper canaliculi was expressed and examined microscopically.

113
ANSWER â•… Case 57

Diagnosis:╇ Canaliculitis secondary to Actinomyces.

Clinical description:╇ Granular calcified material is being expressed from the upper punctum.

Histological description:╇ Clusters of Gram-positive filaments are seen in a matrix of calcified mineralized
tissue.

Collections of Gram-positive
filaments consistent with
Actinomyces
Mineralization

114
Chapter 2â•… Orbit and optic nerve CASE 58 â•…

Courtesy of Jason A. Sokol, MD. Director of Oculofacial Plastic and Orbital Surgery at Kansas
University Eye Center.

A 27-year-old diabetic man presented with right eyelid swelling and a frozen globe.

115
ANSWER â•… Case 58

Diagnosis:╇ Orbital infection with mucormycosis.

Clinical description:╇ Swelling of the upper and lower eyelids with pigmented discharge is noted on clinical
examination.

Histological description:╇ Nonseptate filamentous fungi consistent with Mucor are seen (arrows).

116
Chapter 2â•… Orbit and optic nerve CASE 59 â•…

A 37-year-old man presented with epiphora, swelling, and pain in the left medial canthus area. He
underwent a dacryocystorhinostomy.

117
ANSWER â•… Case 59

Diagnosis:╇ Dacryocystitis.

Clinical description:╇ Clinical examination revealed redness and swelling along the left medial canthus. The
overlying skin is ulcerated.

Histological description:╇ Histopathology reveals tissue lined by nonkeratinized, pseudostratified columnar


epithelium with mucus glands. Chronic nongranulomatous inflammation consisting of lymphocytes and
plasma cells is seen infiltrating the stroma (arrowheads). Gram stain reveals small clusters of Gram-positive
cocci (not shown).

Nonkeratinized pseudostratified
columnar epithelium

Stroma

118
Chapter 2â•… Orbit and optic nerve CASE 60 â•…

Printed with permission from Azari A.A., Kanavi M.R., Saipe N., Lee V., Lucarelli M., Potter H.D., Albert D.M. Transitional Cell
Carcinoma of the Lacrimal Sac Presenting With Bloody Tears. JAMA Ophthalmology 2013; 131(5):689–690.

A 54-year-old man presented with swelling of the right lower eyelid and bloody tears for the past 6
months.

Printed with permission from Azari A.A., Kanavi M.R., Saipe N., Lee V., Lucarelli M., Potter H.D., Albert D.M. Transitional Cell Carcinoma of the
Lacrimal Sac Presenting With Bloody Tears. JAMA Ophthalmology 2013; 131(5):689–690.

119
ANSWER â•… Case 60

Diagnosis:╇ Transitional cell carcinoma of the lacrimal sac.

Clinical description:╇ The CT scan shows a heterogeneous, well-circumscribed mass in the right lacrimal sac
fossa. Intraoperative photograph reveals a large mass within the lacrimal sac.

Histological description:╇ Histopathological examination reveals papillary proliferation of transitional cells


(left). Nuclear pleomorphism and few mitotic figures are noted (center). Immunohistochemistry for HPV-16 is
diffusely positive, suggesting HPV involvement (right).

120
Chapter 2â•… Orbit and optic nerve CASE 61 â•…

A 58-year-old man presented with a 6-month history of double vision. He later underwent an orbital
biopsy.

121
ANSWER â•… Case 61

Diagnosis:╇ Orbital lymphoma (small B-lymphocytic lymphoma).

Clinical description:╇ CT scan shows infiltration of the right orbit. The tumor appears to be conforming to the
lateral orbital wall.

Histological description:╇ Histopathology demonstrates dense infiltrate of small, round, uniform, basophilic
cells. Immunohistochemistry was strongly positive for CD20 (not shown).

122
Chapter 2â•… Orbit and optic nerve CASE 62 â•…

From Albert, Daniel M., Miller, Joan W., Azar, Dimitri T., and Blodi, Barbara A. (eds). 2008. Albert
& Jakobiec’s Principles and Practice of Ophthalmology, 3rd ed. Philadelphia: Copyright Elsevier 2008.

An 85-year-old woman with a history of breast cancer presented with diplopia and a “smaller left eye.”

123
ANSWER â•… Case 62

Diagnosis:╇ Breast cancer with orbital cancer.

Clinical description:╇ Clinical examination demonstrates left eye enophthalmos and narrowing of the
palpebral fissure.

Histological description:╇ Histopathology reveals tissue infiltrated with epithelial cells in an “Indian-file”
arrangement (arrows). These cells are round with eosinophilic cytoplasm and round to irregular nuclei
displaying moderate atypia, including pleomorphism, an enlarged nucleus-to-cytoplasm ratio, and prominent
nucleoli. There are occasional areas of duct formation (asterisks).

* *

124
Chapter 2â•… Orbit and optic nerve CASE 63 â•…

A 37-year-old obese woman presented with decreased vision and chronic headaches.

125
ANSWER â•… Case 63

Diagnosis:╇ Papilledema due to idiopathic intracranial hypertension.

Clinical description:╇ Dilated fundus examination shows swelling of the optic nerve head surrounded by
hemorrhages.

Histological description:╇ Histopathology demonstrates swelling and anterior displacement of the optic
nerve head.

Vitreous
Anteriorly displaced
optic nerve head

Sclera
Retina

rosa
na crib
Choroid Lami

Optic nerve

126
Chapter 2â•… Orbit and optic nerve CASE 64 â•…

A 29-year-old man was referred for evaluation of optic nerve edema.

127
ANSWER â•… Case 64

Diagnosis:╇ Optic nerve drusen.

Clinical description:╇ Close inspection of the optic nerve head reveals deposition of hyaline material within
the optic nerve head.

Histological description:╇ Calcified deposits are seen within the optic nerve head anterior to the lamina
cribrosa.

Drusen

Lamina cribrosa
Sclera

Sclera

Optic nerve

128
Chapter 2â•… Orbit and optic nerve CASE 65 â•…

A 32-year-old woman presented with a sudden onset of decreased vision and the above MRI findings.

129
ANSWER â•… Case 65

Diagnosis:╇ Optic neuritis.

Clinical description:╇ Magnetic resonance imaging demonstrates enhancement of the left optic nerve
consistent with optic neuritis.

Histological description:╇ Histopathology demonstrates a cross-sectional view of the optic nerve. An area of
lymphoplasmacytic inflammation is seen within the optic nerve near a central retinal vessel.

Dura

Optic nerve

Central retinal vessel

Inflammation

Subarachnoid space

130
Chapter 2â•… Orbit and optic nerve CASE 66 â•…

From Albert, Daniel M., Miller, Joan W., Azar, Dimitri T., and Blodi,
Barbara A. (eds). 2008. Albert & Jakobiec’s Principles and Practice of
Ophthalmology, 3rd ed. Philadelphia: Copyright Elsevier 2008.

A 47-year-old woman presented with mild blurring of the vision in her right eye.

131
ANSWER â•… Case 66

Diagnosis:╇ Optic nerve sheath meningioma.

Clinical description:╇ Axial CT scan imaging shows calcification of the optic nerve posteriorly and the classic
“tram-tracking” appearance anteriorly.

Histological description:╇ Histopathology demonstrates proliferation of meningothelial cells surrounding


the optic nerve. Many calcified psammoma bodies are present (asterisks).

Dura mater



∗ ∗ Proliferating meningothelial cells

Subarachnoid space

Optic nerve

132
Chapter 2â•… Orbit and optic nerve CASE 67 â•…

A 14-year-old girl presented with progressive proptosis over the past 9 months.

133
ANSWER â•… Case 67

Diagnosis:╇ Optic nerve glioma.

Clinical description:╇ Left eye proptosis and restricted extraocular motility is noted.

Histological description:╇ The tumor demonstrates proliferation of pilocytic glial cells and astrocytes
(arrowhead). Rosenthal fibers (eosinophilic bundles of fibrils), which are pathognomonic for this condition
are seen.

Rosenthal fibers

134
Chapter 3â•… Cornea and conjunctiva CASE 68 â•…

A 41-year-old woman presented with a longstanding history of corneal findings, as shown above.

135
ANSWER â•… Case 68

Diagnosis:╇ Lattice dystrophy.

Clinical description:╇ The slit-lamp exam demonstrates linear bands of corneal opacification.

Histological description:╇ Histopathology reveals amorphous deposits that stain positive with Congo red
(previous page) and show birefringence (green and orange) under the polarized light.

136
Chapter 3â•… Cornea and conjunctiva CASE 69 â•…

Courtesy of Neal P. Barney, M.D. University of Wisconsin-Madison.

A 23-year-old man presented with corneal opacities, as shown above. He later underwent a
penetrating keratoplasty.

137
ANSWER â•… Case 69

Diagnosis:╇ Granular dystrophy.

Clinical description:╇ Anterior stromal opacities are seen with clear intervening spaces. The corneal opacity
does not involve the peripheral cornea.

Histological description:╇ Staining with Masson trichrome demonstrates subepithelial as well as anterior
stromal corneal infiltrates (arrows). The deposits are composed of hyaline and stain red with Masson
trichrome.

Epithe
lium

Stroma

138
Chapter 3â•… Cornea and conjunctiva CASE 70 â•…

Courtesy of Neal P. Barney, M.D. University of Wisconsin-Madison.

A 32-year-old woman presented with decreased vision. She later underwent penetrating keratoplasty.

139
ANSWER â•… Case 70

Diagnosis:╇ Avellino corneal dystrophy.

Clinical description:╇ Slit-lamp examination reveals multiple confluent well-circumscribed stromal opacities.

Histological description:╇ The histopathology has features of both lattice and granular dystrophy. PAS stain
deposits (arrow, top) that also stain positive with Congo red and demonstrate dichroism under the polarized
light (not shown), typical of lattice dystrophy. Masson trichrome staining shows hyaline deposits in the
anterior stroma, as seen in granular dystrophy (bottom). Hyaline deposits are shown with arrows.

Epithelium

Stroma

Epithelium

Stroma

140
Chapter 3â•… Cornea and conjunctiva
71
CASE
â•…

Courtesy of Neal P. Barney, M.D. University of Wisconsin-Madison.

A 19-year-old patient presented with decreased vision and photophobia.

141
ANSWER â•… Case 71

Diagnosis:╇ Macular dystrophy.

Clinical description:╇ Confluent corneal opacities extending to the limbus without clear intervening spaces
are seen on the slit-lamp examination.

Histological description:╇ Infiltrates are seen at all levels. Alcian blue demonstrates positive staining of the
mucopolysaccharide deposits.

Corneal deposits

Corneal deposits

142
Chapter 3â•… Cornea and conjunctiva CASE 72 â•…

From Albert, Daniel M., Miller, Joan W., Azar, Dimitri T., and Blodi, Barbara A. (eds). 2008. Albert
& Jakobiec’s Principles and Practice of Ophthalmology, 3rd ed. Philadelphia: Copyright Elsevier 2008.

A 38-year-old woman with a history of ocular pain, and foreign body sensation, worse in the
morning, presented with the above slit-lamp examination finding.

143
ANSWER â•… Case 72

Diagnosis:╇ Anterior basement membrane dystrophy.

Clinical description:╇ The slit-lamp examination demonstrates the typical “map” dystrophy within the
anterior cornea.

Histological description:╇ Histopathology demonstrates reduplicated epithelial basement membrane. There


is also separation of the epithelial basement membrane from the underlying Bowman’s layer in some areas
(arrowhead).

Corneal epithelium

Reduplicated epithelial
basement membrane

Bowman’s layer

Stroma

144
Chapter 3â•… Cornea and conjunctiva CASE 73 â•…

Courtesy of Neal P. Barney, M.D. University of Wisconsin-Madison.

A 56-year-old patient with a past ocular history of longstanding retinal detachment presented with
decreased vision and ocular pain.

145
ANSWER â•… Case 73

Diagnosis:╇ Band keratopathy.

Clinical description:╇ Cornea is deposited with plaques of calcified material.

Histological description:╇ von Kossa stain demonstrates calcium deposits within the epithelial basement
membrane and Bowman’s layer. The surface epithelium is attenuated and absent in some areas.

ts
e p osi an’s
d m er
l c ium e Bow ial lay
Ca in th ithel
h p
wit sube
a n d

eal
c orn
ce m
rfa eliu
Su epith

a
st rom
rneal
Co

146
Chapter 3â•… Cornea and conjunctiva CASE 74 â•…

A 6-year-old patient with bilateral corneal opacity underwent penetrating keratoplasty and
conjunctival biopsy.

147
ANSWER â•… Case 74

Diagnosis:╇ Cystinosis.

Clinical description:╇ Clinical examination reveals punctate areas of opacity within the corneal stroma.

Histological description:╇ Histopathology reveals goblet cells containing nonkeratinized stratified squamous
epithelium. Cystine crystals (circles) are seen within the conjunctival stroma.

Goblet cells
Conjunctival epithelium

Conjunctival stroma

148
Chapter 3â•… Cornea and conjunctiva CASE 75 â•…

From Albert, Daniel M., Miller, Joan W., Azar, Dimitri T., and Blodi, Barbara A. (eds). 2008. Albert & Jakobiec’s Principles and Practice of
Ophthalmology, 3rd ed. Philadelphia: Copyright Elsevier 2008.

A 43-year-old patient presented with dry eyes and foreign body sensation.

149
ANSWER â•… Case 75

Diagnosis:╇ Salzmann’s nodular degeneration.

Clinical description:╇ The slit-lamp examination reveals multiple bluish-gray elevated anterior corneal
nodules.

Histological description:╇ Examination of the histopathological sections reveals thick, fibrous tissue
(asterisks) interposed between Bowman’s membrane and the irregular epithelium.

Epithelium Bowman’s
membrane

Stroma

150
Chapter 3â•… Cornea and conjunctiva CASE 76 â•…

From Albert, Daniel M., Miller, Joan W., Azar, Dimitri T., and Blodi,
Barbara A. (eds). 2008. Albert & Jakobiec’s Principles and Practice of
Ophthalmology, 3rd ed. Philadelphia: Copyright Elsevier 2008.

A 53-year-old man presented with bilateral decreased vision with the above external appearance.
He had similar findings in his other eye.

151
ANSWER â•… Case 76

Diagnosis:╇ Spheroidal degeneration.

Clinical description:╇ The slit-lamp photograph demonstrates many spheroidal deposits within the stroma of
the cornea.

Histological description:╇ Sections through the specimen show amorphous deposits of varying size in the
anterior stroma and epithelium (arrows) that are markedly positive with elastin stain (bottom). Extensive
subepithelial fibrosis is seen. The epithelium is of variable thickness and shows intracellular edema.

Epithelium

Stromal fibrosis

152
Chapter 3â•… Cornea and conjunctiva CASE 77 â•…

Courtesy of Neal P. Barney, M.D. University of Wisconsin-Madison.

A 25-year-old man presented with decreased vision and the above ophthalmic examination finding.

153
ANSWER â•… Case 77

Diagnosis:╇ Keratoconus.

Clinical description:╇ Cone-shape deformity of the cornea along with the classic Munson’s sign (V-shaped
deformity of the lower eyelid in downward gaze) is demonstrated.

Histological description:╇ Histopathology demonstrates a thinned cornea with attenuated epithelium.


Multiple breaks (arrowheads) within the Bowman’s layer are seen. The endothelium is well preserved.

Bowman’s layer
Attenuated epithelium

Stromal thinning
and scarring

Well-preserved
endothelium

154
Chapter 3â•… Cornea and conjunctiva CASE 78 â•…

A 72-year-old man presented with dry eyes and the above clinical picture.

155
ANSWER â•… Case 78

Diagnosis:╇ Corneal dellen.

Clinical description:╇ Slit-lamp examination reveals corneal thinning just inside the limbus.

Histological description:╇ Histopathology reveals corneal thinning with artifactuous breaks in the Descemet’s
membrane.

Epithelial attenuation

Stroma

Descemet’s
membrane

156
Chapter 3â•… Cornea and conjunctiva CASE 79 â•…

Courtesy of Neal P. Barney, M.D. University of Wisconsin-Madison.

A 55-year-old man presented with poor vision, which could not be improved with refraction.

157
ANSWER â•… Case 79

Diagnosis:╇ Keratoglobus.

Clinical description:╇ The slit-lamp examination shows generalized thinning and globular protrusion of the
cornea.

Histological description:╇ The cornea is thin centrally as well as peripherally. The epithelium is attenuated
with loss of Bowman’s layer in some areas.

Severely attenuated
epithelium

Loss of Bowman’s
layer

158
Chapter 3â•… Cornea and conjunctiva CASE 80 â•…

Courtesy of Neal P. Barney, M.D. University of Wisconsin-Madison.

A 69-year-old woman presented with decreased vision, worse upon waking up. After failing medical
therapy, she underwent a penetrating keratoplasty.

159
ANSWER â•… Case 80

Diagnosis:╇ Fuchs’ endothelial dystrophy.

Clinical description:╇ Slit-lamp examination reveals marked corneal edema with a “beaten bronze”
appearance to the posterior cornea.

Histological description:╇ An edematous cornea with loss of normal stromal cleft is seen. Anvil-shaped
excrescences consistent with guttae are seen.

Edematous cornea

Guttae
Thickening of the
Descemet’s membrane

160
Chapter 3â•… Cornea and conjunctiva CASE 81 â•…

From Albert, Daniel M., Miller, Joan W., Azar, Dimitri T., and Blodi, Barbara A. (eds). 2008. Albert
& Jakobiec’s Principles and Practice of Ophthalmology, 3rd ed. Philadelphia: Copyright Elsevier 2008.

A 16-month-old infant presented with decreased vision and nystagmus.

161
ANSWER â•… Case 81

Diagnosis:╇ Congenital hereditary endothelial dystrophy (CHED).

Clinical description:╇ Slit-lamp examination reveals mild corneal haze.

Histological description:╇ Histopathology reveals a thickened cornea with loss of natural stromal clefts. The
Descemet’s membrane is thick and the endothelial cell layer is absent.

Corneal epithelium

Corneal stromal edema with


increased thickness

Descemet’s membrane

162
Chapter 3â•… Cornea and conjunctiva CASE 82 â•…

Courtesy of Neal P. Barney, M.D. University of Wisconsin-Madison.

A 4-year-old girl with decreased vision since birth presented with the above corneal finding on
slit-lamp examination. She later underwent a penetrating keratoplasty.

163
ANSWER â•… Case 82

Diagnosis:╇ Posterior polymorphous dystrophy.

Clinical description:╇ Slit-lamp examination reveals multiple scalloped corneal opacities located posteriorly.

Histological description:╇ Histopathology demonstrates a multilayer endothelium.

Corneal stroma

Descemet’s membrane

Multilayered
endothelium

164
Chapter 3â•… Cornea and conjunctiva CASE 83 â•…

A 32-year-old man with no ocular complaints presented for a routine eye examination.

165
ANSWER â•… Case 83

Diagnosis:╇ Hassall–Henle bodies.

Clinical description:╇ A slit-lamp examination reveals a normal cornea.

Histological description:╇ Histopathology demonstrates peripheral guttae in this otherwise normal cornea.

Corneal stroma

Descemet’s
membrane

Guttae

166
Chapter 3â•… Cornea and conjunctiva CASE 84 â•…

From Albert, Daniel M., Miller, Joan W., Azar, Dimitri T., and Blodi, Barbara A. (eds). 2008. Albert
& Jakobiec’s Principles and Practice of Ophthalmology, 3rd ed. Philadelphia: Copyright Elsevier 2008.

A 77-year-old terminally ill patient was seen as an inpatient with the above slit-lamp examination
finding.

Courtesy of Neal P. Barney, M.D. University of Wisconsin-Madison.

167
ANSWER â•… Case 84

Diagnosis:╇ Wilson’s disease.

Clinical description:╇ Slit-lamp examination reveals a pigmented Descemet’s membrane consistent with
Kayser–Fleischer ring.

Histological description:╇ Histopathology demonstrates a dark band of copper deposit on the posterior
aspect of the Descemet’s membrane (short arrows).

Descemet’s membrane
Endothelium Stroma

Courtesy of Neal P. Barney, M.D. University of Wisconsin-Madison.

168
Chapter 3â•… Cornea and conjunctiva CASE 85 â•…

A 2-year-old boy with a history of Goldenhar syndrome presented with a limbal mass.

169
ANSWER â•… Case 85

Diagnosis:╇ Limbal dermoid.

Clinical description:╇ Slit-lamp examination reveals an elevated yellow mass with associated cilia straddling
the temporal limbus.

Histological description:╇ Histopathology reveals tissue lined by keratinized stratified squamous epithelium
with associated pilosebaceous units. The underlying dermis is fibrotic.

Keratinized stratified
squamous epithelium

Fibrotic dermis

Sebaceous gland Pilar units

170
Chapter 3â•… Cornea and conjunctiva CASE 86 â•…

Courtesy of Neal P. Barney, M.D. University of Wisconsin-Madison.

A 63-year-old woman with a history of rheumatoid arteritis presented with central corneal thinning.

171
ANSWER â•… Case 86

Diagnosis:╇ Descemetocele.

Clinical description:╇ Slit-lamp examination shows a central area of corneal thinning consistent with
Descemetocele.

Histological description:╇ Histopathology shows complete loss of the stroma centrally. Only Descemet’s
membrane and an attenuated epithelium remain in this area.

Epithelium

Epithelium
Stroma

Descemet’s membrane
Descemet’s membrane

172
Chapter 3â•… Cornea and conjunctiva CASE 87 â•…

A 32-year-old contact lens wearer presented with a longstanding central corneal ulcer. The ulcer was
treated with fortified antibiotic eye drops with no response. Corneal cultures were negative for
bacteria, fungus, and acanthamoeba. Therapeutic penetrating keratoplasty was performed.

173
ANSWER â•… Case 87

Diagnosis:╇ Acanthamoeba keratitis.

Clinical description:╇ Central corneal infiltrate is seen. There are multiple satellite lesions surrounding the
main infiltrate. The remaining cornea is edematous.

Histological description:╇ The corneal tissue is infiltrated with round, double wall cysts of acanthamoeba
(arrows). Infiltration with neutrophils is also seen.

174
Chapter 3â•… Cornea and conjunctiva CASE 88 â•…

Courtesy of Sarah M. Nehls, M.D. University of Wisconsin-Madison.

A 63-year-old farmer presented with a corneal ulcer and hypopyon. The condition worsened despite
topical antifungal therapy. He subsequently underwent a penetrating keratoplasty.

175
ANSWER â•… Case 88

Diagnosis:╇ Fungal keratitis secondary to Aspergillus.

Clinical description:╇ Slit-lamp examination reveals a central corneal infiltrate with feathery edges. A 3-mm
hypopyon is present.

Histological description:╇ Gomori methenamine silver (GMS) stain demonstrates septated branching
filamentous fungi. The branching is at 45-degree angles (circles) consistent with Aspergillus.

176
Chapter 3â•… Cornea and conjunctiva CASE 89 â•…

A 36-year-old man presented with ocular pain and photophobia. The slit-lamp examination is seen
above. Later he developed corneal scarring and decreased vision.

177
ANSWER â•… Case 89

Diagnosis:╇ Herpes simplex virus (HSV) keratitis.

Clinical description:╇ Corneal dendritic lesions with terminal bulbs are seen.

Histological description:╇ A marked inflammatory response in the posterior cornea is present. The
Descemet’s membrane is fragmented. Epithelioid and multinucleated giant cells are also present in the area of
Descemet’s membrane.

Marked inflammation in the posterior


corneal stroma

Multinucleated giant cell

Fragmented Descemet’s membrane

178
Chapter 3â•… Cornea and conjunctiva CASE 90 â•…

From Albert, Daniel M., Miller, Joan W., Azar, Dimitri T., and Blodi, Barbara A. (eds). 2008. Albert
& Jakobiec’s Principles and Practice of Ophthalmology, 3rd ed. Philadelphia: Copyright Elsevier 2008.

A 42-year-old man with AIDS presented with decreased vision and corneal changes.

179
ANSWER â•… Case 90

Diagnosis:╇ Microsporidial keratitis.

Clinical description:╇ Slit-lamp examination demonstrates multiple punctate corneal opacities.

Histological description:╇ Histopathological evaluation of the corneal tissue demonstrates anterior stromal
deposits consistent with microsporidia (arrows).

Bowman’s layer

Stroma

180
Chapter 3â•… Cornea and conjunctiva CASE 91 â•…

A 53-year-old man with a history of corneal transplant presented with a corneal infiltrate not
responding to topical antibiotics. He had several previous penetrating keratoplasties.

181
ANSWER â•… Case 91

Diagnosis:╇ Bacterial keratitis due to MRSA (methicillin resistant Staphylococcus aureus).

Clinical description:╇ Slit-lamp examination demonstrates a central corneal infiltrate with an overlying
epithelial defect.

Histological description:╇ Corneal tissue is infiltrated with numerous neutrophils (top). Gram stain
demonstrates scattered clusters of Gram-positive cocci (arrows).

Corneal inflammation

182
Chapter 3â•… Cornea and conjunctiva CASE 92 â•…

Printed with permission from Azari A.A., Nehls S., Ghoghawala S.Y., Lee V., Kanavi M.R., Potter
H.D. Rhizopus Keratitis Following Corneal Trauma. JAMA Ophthalmology 2013; 131(6):776.

A 48-year-old man presented with a central corneal ulcer after trauma with a metal wire.

Printed with permission from Azari A.A., Nehls S., Ghoghawala S.Y., Lee V., Kanavi M.R., Potter H.D. Rhizopus Keratitis Following Corneal Trauma.
JAMA Ophthalmology 2013; 131(6):776.

183
ANSWER â•… Case 92

Diagnosis:╇ Rhizopus keratitis.

Clinical description:╇ Slit-lamp examination shows a central necrotic corneal ulcer with an overlying
epithelial defect.

Histological description:╇ Branching filamentous fungi are seen demonstrating an exquisite pattern of
autofluorescence (top). Microscopic evaluation demonstrates multiple branching nonseptate hyphae
(arrows, bottom).

Printed with permission from Azari A.A., Nehls S., Ghoghawala S.Y., Lee V., Kanavi
M.R., Potter H.D. Rhizopus Keratitis Following Corneal Trauma. JAMA Ophthalmology
2013; 131(6):776.

184
Chapter 3â•… Cornea and conjunctiva CASE 93 â•…

Courtesy of Neal P. Barney, M.D. University of Wisconsin-Madison.

A 43-year-old woman with past history of ocular shingles presented with a nonhealing corneal ulcer.

185
ANSWER â•… Case 93

Diagnosis:╇ Herpes zoster keratitis.

Clinical description:╇ Nonhealing corneal ulcer.

Histological description:╇ Histopathology reveals central corneal thinning (double headed arrow) with
marked inflammation. An overlying epithelial defect, consistent with neurotrophic ulcer, is present.

Corneal epithelium

Stroma

186
Chapter 3â•… Cornea and conjunctiva CASE 94 â•…

Courtesy of Neal P. Barney, M.D. University of Wisconsin-Madison.

A 45-year-old woman presented with decreased vision and corneal neovascularization as shown above.

187
ANSWER â•… Case 94

Diagnosis:╇ Interstitial keratitis.

Clinical description:╇ Slit-lamp examination demonstrates a round area of corneal opacity with
neovascularization.

Histological description:╇ Marked inflammation, vascularization, and scarring are seen in the posterior third
of the cornea.

Scarring and
inflammation

Blood vessels

188
Chapter 3â•… Cornea and conjunctiva CASE 95 â•…

Courtesy of Matthew J. Thompson, Tower Clock Eye Center, Green Bay,


WI.

A 77-year-old man presented with corneal neovascularization and opacity unresponsive to topical and
subconjunctival steroids. He subsequently underwent an enucleation.

189
ANSWER â•… Case 95

Diagnosis:╇ Corneal squamous cell carcinoma.

Clinical description:╇ Examination with the slit-lamp demonstrates corneal scar and whitening.

Histological description:╇ Histopathology shows an edematous cornea infiltrated with epithelioid cells with
severe atypia (arrows, top) staining with cytokeratin AE1/AE3, a marker for squamous cells (arrows, bottom).

190
Chapter 3â•… Cornea and conjunctiva CASE 96 â•…

A 65-year-old man presented with the above corneal changes 6 months after a complicated cataract
surgery.

191
ANSWER â•… Case 96

Diagnosis:╇ Epithelial ingrowth.

Clinical description:╇ Epithelial ingrowth causing corneal haze is evident. The leading edge of the epithelial
ingrowth has scalloped borders.

Histological description:╇ Growth of nonkeratinized stratified squamous epithelium on the posterior surface
of the cornea as well as anterior iris is shown.

Epithelial ingrowth

Iris stroma Iris dilator Posterior pigmented iris


muscle epithelium

192
Chapter 3â•… Cornea and conjunctiva CASE 97 â•…

From Albert, Daniel M., Miller, Joan W., Azar, Dimitri T., and Blodi, Barbara A. (eds). 2008. Albert
& Jakobiec’s Principles and Practice of Ophthalmology, 3rd ed. Philadelphia: Copyright Elsevier 2008.

A 68-year-old man with a history of Fuchs’ endothelial dystrophy was seen in the clinic on
postoperative day 1, after a Descemet’s stripping endothelial keratoplasty (DSEK).

193
ANSWER â•… Case 97

Diagnosis:╇ Corneal button demonstrating a failed DSEK procedure obtained after full thickness corneal
transplant.

Clinical description:╇ Slit-lamp examination reveals an eye after DSEK surgery with graft in a good central
position. A small air bubble remains superiorly, but usually resolves in 2–3 days.

Histological description:╇ Histopathology demonstrates a traditional full thickness corneal transplant


performed after failed DSEK surgery. Host Descemet’s membrane is present at the donor–recipient interface.

Host epithelium Retained host Descemet’s


membrane at the interface
Host cornea

Donor cornea

Donor Descemet’s
membrane

194
Chapter 3â•… Cornea and conjunctiva CASE 98 â•…

Courtesy of Sarah M. Nehls, M.D. University of Wisconsin-Madison.

A 43-year-old man with progressive keratoconus presented with the above clinical picture 1 month
postoperatively. Many years later, he required a corneal transplant.

195
ANSWER â•… Case 98

Diagnosis:╇ Intrastromal corneal ring segments (INTACS) in an eye with a history of keratoconus.

Clinical description:╇ Slit-lamp examination demonstrates two INTACS placed superiorly and inferiorly.

Histological description:╇ Histopathology demonstrates a thinned cornea with an intrastromal defect where
the INTACS were placed (asterisk).


Epithelium

Stroma

196
Chapter 3â•… Cornea and conjunctiva CASE 99 â•…

Courtesy of Neal P. Barney, M.D. University of Wisconsin-Madison.

A 48-year-old male patient has a longstanding history of corneal edema and decreased vision no
longer amenable to medical therapy. He underwent penetrating keratoplasty.

197
ANSWER â•… Case 99

Diagnosis:╇ Pseudophakic bullous keratopathy.

Clinical description:╇ Slit-lamp examination demonstrates marked corneal haze and edema.

Histological description:╇ The cornea is thickened. Areas of separation between the epithelium and the
Bowman’s layer are seen (asterisk).

Surface epithelium

Increase in
corneal thickness
Corneal stroma

198
Chapter 3â•… Cornea and conjunctiva CASE 100 0â•…

Courtesy of Neal P. Barney, M.D. University of Wisconsin-Madison.

A 48-year-old man presented with decreased vision and photophobia 2 years following a corneal
transplant.

199
ANSWER â•… Case 100

Diagnosis:╇ Failed corneal graft.

Clinical description:╇ A hazy and edematous cornea with areas of neovascularization is seen.

Histological description:╇ The corneal tissue is edematous with loss of natural stromal clefts. Areas of
inflammation, particularly at the edges of the transplant, are apparent.

Peripheral
Thickened cornea inflammation
Peripheral with loss of
inflammation natural clefts

200
Chapter 3â•… Cornea and conjunctiva CASE 101 â•…

A 48-year-old man presented with irregular astigmatism uncorrected with rigid gas permeable lenses.
He underwent penetrating keratoplasty.

201
ANSWER â•… Case 101

Diagnosis:╇ Previous radial keratotomy.

Clinical description:╇ Multiple radially oriented linear corneal scars consistent with radial keratotomy are seen.

Histological description:╇ Histopathology demonstrates multiple partial thickness corneal scars (arrowheads).

202
Chapter 3â•… Cornea and conjunctiva CASE 102 â•…

A 54-year-old male presented with corneal clouding 2 months after injury with a softball. The corneal
clouding did not resolve and he underwent a penetrating keratoplasty.

203
ANSWER â•… Case 102

Diagnosis:╇ Corneal blood staining.

Clinical description:╇ Corneal blood staining and opacification are seen.

Histological description:╇ Red blood cells are seen scattered within an edematous cornea.

Thickened
cornea

204
Chapter 3â•… Cornea and conjunctiva CASE 103 â•…

The patient is a 20-year-old man with history of chemical burn of the left eye from propionic acid.
He underwent stem cell transplant. The specimen below was removed from his corneal surface.

205
ANSWER â•… Case 103

Diagnosis:╇ Corneal conjunctivalization secondary to stem cell deficiency.

Clinical description:╇ Clinical examination reveals corneal haze and pannus secondary to stem cell deficiency.

Histological description:╇ Histopathology reveals goblet cells containing nonkeratinized stratified squamous
epithelium of the conjunctiva overlying a loose, highly vascularized (asterisks) substantia propria with
lymphoplasmacytic infiltrate (arrowheads).

Goblet cells within the conjunctival


epithelium

*
* * *
Conjunctival stroma

206
Chapter 3â•… Cornea and conjunctiva CASE 104 â•…

A 38-year-old man presented with tearing, redness, and pain in his left eye after working in his garage.

207
ANSWER â•… Case 104

Diagnosis:╇ Corneal foreign body.

Clinical description:╇ A foreign body with a surrounding rust ring is seen embedded within the cornea.

Histological description:╇ The cornea demonstrates profound scarring and inflammation. There is loss of
keratocytes and stromal disorganization within the anterior two-thirds of the cornea. Foreign body material
is noted.

Corneal scarring and disorganization

Corneal foreign body

Inflammation

208
Chapter 3â•… Cornea and conjunctiva CASE 105 â•…

Courtesy of Neal P. Barney, M.D. University of Wisconsin-Madison.

A 12-year-old boy with a history of forceps delivery presented with the above corneal finding. He
underwent a penetrating keratoplasty many years later.

209
ANSWER â•… Case 105

Diagnosis:╇ Vertical breaks in the Descemet’s membrane in a patient with a history of forceps delivery.

Clinical description:╇ Vertical breaks in the Descemet’s membrane are noted.

Histological description:╇ Histopathology reveals a scarred corneal stroma. The surface epithelium is largely
missing. There is a break in the Descemet’s membrane with scrolling of this membrane.

Scrolled up edges of
Descemet’s membrane

210
Chapter 3â•… Cornea and conjunctiva CASE 106 â•…

A 42-year-old woman presented with a new conjunctival lesion as shown above. She underwent
excisional biopsy of her conjunctival lesion.

211
ANSWER â•… Case 106

Diagnosis:╇ Primary acquired melanosis (PAM) without atypia.

Clinical description:╇ A flat pigmented lesion on the superior bulbar conjunctiva is seen.

Histological description:╇ Histopathology reveals conjunctiva with mild acanthosis. There is an increased
number of bland-appearing pigmented melanocytes along the basal epithelial layer (arrowheads). There is no
atypia, no melanocytic nesting nor intraepithelial spread.

Goblet cells

Conjunctival epithelium

Conjunctival stroma

Pigment-laden macrophages

212
Chapter 3â•… Cornea and conjunctiva CASE 107 â•…

From Albert, Daniel M., Miller, Joan W., Azar, Dimitri T., and Blodi, Barbara A. (eds) 2008. Albert
& Jakobiec’s Principles and Practice of Ophthalmology, 3rd ed. Philadelphia: Copyright Elsevier 2008.

A 49-year-old woman presented with darkly pigmented conjunctival lesions as shown above. She later
underwent conjunctival biopsy.

213
ANSWER â•… Case 107

Diagnosis:╇ Benign acquired melanosis (BAM), also known as racial melanosis.

Clinical description:╇ The clinical examination reveals multiple pigmented conjunctival lesions.

Histological description:╇ Histopathology demonstrates tissue lined by nonkeratinized stratified squamous


epithelium with an increased number of bland-appearing pigmented melanocytes along the basal epithelial
layer (arrows).

Conjunctival epithelium

Conjunctival stroma

214
Chapter 3â•… Cornea and conjunctiva CASE 108 â•…

A 48-year-old Caucasian woman presented with the pigmented lesions as shown above.

215
ANSWER â•… Case 108

Diagnosis:╇ Primary acquired melanosis (PAM) with atypia.

Clinical description:╇ Conjunctival pigmentation is seen in the lateral limbus spanning an area
approximating four clock hours.

Histological description:╇ Histopathological evaluation demonstrates involvement of the superficial


(arrows), as well as deep (arrowheads) epithelium, with pigmented melanocytes.

216
Chapter 3â•… Cornea and conjunctiva CASE 109 â•…

A 14-year-old girl presented with a slow-growing conjunctival lesion. The lesion has continued to
enlarge in recent years.

217
ANSWER â•… Case 109

Diagnosis:╇ Conjunctival nevus.

Clinical description:╇ A raised, cystic, pigmented bulbar conjunctival lesion is seen on clinical examination.

Histological description:╇ Histopathology reveals a nonkeratinized stratified squamous epithelium. Nests of


melanocytes (arrowheads) are seen below the epithelium. Multiple epithelial-lined cysts are also present
(asterisks).

Nonkeratinized stratified
squamous epithelium

* *

218
Chapter 3â•… Cornea and conjunctiva CASE 110 â•…

An 84-year-old man presented with a 2-year history of a left caruncular lesion. He underwent
excisional biopsy of the lesion.

219
ANSWER â•… Case 110

Diagnosis:╇ Caruncular nevus.

Clinical description:╇ Slit-lamp examination reveals a pigmented caruncular lesion.

Histological description:╇ The slides reveal nonkeratinized stratified squamous epithelium with goblet cells
and associated pilosebaceous units. Nests of melanocytic cells with variable pigmentation are seen within the
stroma.

Epithelium Goblet cells

Nests of melanocytic
cells with pigmentation
Pilar unit Sebaceous
gland

220
Chapter 3â•… Cornea and conjunctiva CASE 111 â•…

A 66-year-old man presented with a darkly pigmented conjunctival lesion. An excisional biopsy was
performed.

221
ANSWER â•… Case 111

Diagnosis:╇ Conjunctival melanoma arising from nevus.

Clinical description:╇ Anterior segment examination reveals a raised, darkly pigmented conjunctival lesion
with feeder vessels.

Histological description:╇ The conjunctival epithelium (asterisk) and stroma is infiltrated with melanocytic
cells (arrows) with associated epithelial inclusion cysts. Some of these melanocytic cells are arranged in nests,
while others demonstrate marked pleomorphism and prominent nucleoli.

Epithelial
inclusion
cyst

222
Chapter 3â•… Cornea and conjunctiva CASE 112 â•…

Courtesy of Neal P. Barney, M.D. University of Wisconsin-Madison.

An 87-year-old woman presented with the above clinical picture. Two years earlier the patient was
diagnosed with a skin melanoma in her thumb.

223
ANSWER â•… Case 112

Diagnosis:╇ Metastatic conjunctival melanoma.

Clinical description:╇ Slit-lamp examination demonstrates a large raised conjunctival mass.

Histological description:╇ The conjunctival tissue is infiltrated with clusters of large atypical melanocytes
with prominent nucleoli and pigmentation.

224
Chapter 3â•… Cornea and conjunctiva CASE 113 â•…

Courtesy of Neal P. Barney, M.D. University of Wisconsin-Madison.

A 23-year-old man presented with a 2-year history of a slow growing left eye lesion. He underwent
excisional biopsy of the eyelid lesion.

225
ANSWER â•… Case 113

Diagnosis:╇ Pterygium and nevus occurring in the same area.

Clinical description:╇ Clinical examination reveals a vascularized, balloon-like mass straddling the limbus,
nasally.

Histological description:╇ Histopathological examination demonstrates conjunctival epithelium with solar


elastosis, consistent with a pterygium (left side, asterisk), and proliferation of melanocytic cells in the stroma
and epithelial–stromal interface, consistent with a junctional nevus (right side, arrows).

Conjunctival epithelium

Conjunctival epithelium

* Stroma

226
Chapter 3â•… Cornea and conjunctiva CASE 114 â•…

A 44-year-old Hispanic man presented with a growing lesion in his left eye.

227
ANSWER â•… Case 114

Diagnosis:╇ Pterygium.

Clinical description:╇ The clinical examination reveals a vascular “insect wing-like” growth over the nasal
cornea.

Histological description:╇ Histopathology reveals a lesion with nonkeratinized stratified epithelium. Solar
elastotic changes are present within the stroma (asterisks). A fragment of the cornea is also present.

Corneal epithelium

Conjunctival epithelium

*
* *

228
Chapter 3â•… Cornea and conjunctiva CASE 115 â•…

A 32-year-old man presented with a raised conjunctival lesion.

229
ANSWER â•… Case 115

Diagnosis:╇ Pinguecula.

Clinical description:╇ Clinical examination reveals an elevated lesion with mild leukoplakia.

Histological description:╇ Histopathology demonstrates solar elastotic changes (asterisk) within the
conjunctival stroma.

va
*
ti
nc Sclera
nju
Co

Ciliary body

Cornea

Iris

Conjunctival epithelium

Sclera

230
Chapter 3â•… Cornea and conjunctiva CASE 116 â•…

From Albert, Daniel M., Miller, Joan W., Azar, Dimitri T., and Blodi, Barbara A. (eds). 2008. Albert
& Jakobiec’s Principles and Practice of Ophthalmology, 3rd ed. Philadelphia: Copyright Elsevier 2008.

A 79-year-old woman presented with ocular discomfort and epiphora.

231
ANSWER â•… Case 116

Diagnosis:╇ Oncocytoma.

Clinical description:╇ Slit-lamp examination shows an elevated round caruncular lesion.

Histological description:╇ Low power histopathology (top) reveals tissue lined by goblet-cell-containing
nonkeratinized, stratified squamous epithelium with associated pilosebaceous units. Underlying the surface
epithelium, sheets of epithelial cells forming ductal and glandular structures (arrowheads) that contain PAS
positive material within their lumen are seen (asterisks, bottom). The epithelial cells exhibit a granular
eosinophilic cytoplasm.

Surface epithelium Pilosebaceous units

*
*

232
Chapter 3â•… Cornea and conjunctiva CASE 117 â•…

A 31-year-old man presented with increased redness and discomfort in his right eye. A biopsy was
performed. The hematoxylin and eosin stain is on the left; the Congo red stain is on the right.

233
ANSWER â•… Case 117

Diagnosis:╇ Conjunctival amyloidosis.

Clinical description:╇ The slit-lamp examination demonstrates a yellow elevated lesion on the inferior bulbar
conjunctiva.

Histological description:╇ The slides reveal a specimen covered by conjunctival epithelium. There are
numerous, large amorphous eosinophilic deposits in the stroma (top). The eosinophilic material stains with
Congo red (bottom) and displays dichroism under polarized light (not shown).

Conjunctival epithelium

Large eosinophilic
deposits staining positive
with Congo red

234
Chapter 3â•… Cornea and conjunctiva
118
CASE
â•…

From Albert, Daniel M., Miller, Joan W., Azar, Dimitri T., and Blodi, Barbara A. (eds). 2008. Albert
& Jakobiec’s Principles and Practice of Ophthalmology, 3rd ed. Philadelphia: Copyright Elsevier 2008.

A 7-year-old boy presented with left eye discomfort and excessive tearing.

235
ANSWER â•… Case 118

Diagnosis:╇ Conjunctival squamous papilloma.

Clinical description:╇ Slit-lamp examination demonstrates several papillomatous conjunctival lesions located
nasally.

Histological description:╇ Histopathology reveals tissue lined by nonkeratinized stratified squamous


epithelium containing goblet cells. There are multiple papillary fronds (circles) of acanthotic nonkeratinized
stratified squamous epithelium.

Goblet cells Stroma

236
Chapter 3â•… Cornea and conjunctiva CASE 119 â•…

From Albert, Daniel M., Miller, Joan W., Azar, Dimitri T., and Blodi, Barbara A. (eds). 2008. Albert
& Jakobiec’s Principles and Practice of Ophthalmology, 3rd ed. Philadelphia: Copyright Elsevier 2008.

A 26-year-old Native American man presented with a conjunctival lesion.

237
ANSWER â•… Case 119

Diagnosis:╇ Hereditary benign intraepithelial dyskeratosis (HBID).

Clinical description:╇ A clinical photograph demonstrates a limbal leukoplakic lesion in this patient.

Histological description:╇ Histopathology demonstrates hyperkeratosis, acanthosis, parakeratosis, and


dyskeratosis (keratin within the individual epithelial cells, shown with asterisks in the bottom image).

Hyperkeratosis

* *
Acanthosis
*

* *
*

238
Chapter 3â•… Cornea and conjunctiva CASE 120 â•…

A 44-year-old man presented with a newly-formed lesion in his left eye. The lesion appeared
approximately 3 months ago and it has been increasing in size.

239
ANSWER â•… Case 120

Diagnosis:╇ Conjunctival intraepithelial neoplasia (CIN).

Clinical description:╇ An elevated erythematous lesion with multiple pin-point blood vessels (strawberry
lesion) is seen.

Histological description:╇ Histopathology demonstrates a sharp transition from normal to acanthotic


epithelium. The involved epithelium is markedly acanthotic and loss of polarity is noted. Other features
may include solar elastosis of the stroma and mitotic figures (not shown).

Sharp transition from


normal to acanthotic
epithelium

240
Chapter 3â•… Cornea and conjunctiva CASE 121 â•…

Courtesy of Robert J. Peralta, MD Ophthalmic Plastic & Reconstructive Surgery, Silkiss Eye Surgery.

The patient is a 68-year-old man with a 4-month history of a rapidly growing conjunctival lesion.

241
ANSWER â•… Case 121

Diagnosis:╇ Invasive conjunctival squamous cell carcinoma.

Clinical description:╇ Anterior segment examination reveals an elevated lesion with a large feeder vessel. The
lesion is arising from the conjunctiva and encroaching upon the cornea.

Histological description:╇ There is a loss of polarity in the epithelium with marked atypia and
pleomorphism. Nests of malignant squamous cells arising from the surface epithelium infiltrate the stroma
(asterisks).

elium
ce epith
Surfa

*
* * * *

242
Chapter 3â•… Cornea and conjunctiva CASE 122 â•…

A 57-year-old man presented with conjunctival lesions of an unknown duration. He later underwent
excision of the conjunctival lesions.

243
ANSWER â•… Case 122

Diagnosis:╇ Conjunctival fibrous histiocytoma.

Clinical description:╇ Clinical examination reveals a red, elevated conjunctival lesion.

Histological description:╇ Densely packed sheets of fibroblasts in a collagenous matrix are seen in a storiform
pattern. Vacuolated histiocytes are noted in between the fibroblasts.

244
Chapter 3â•… Cornea and conjunctiva CASE 123 â•…

Printed with permission from Lee V., Azari A.A., Nehls S., Potter H.D.,
Albert D.A. Leiomyoma of the Lower Eyelid. JAMA Ophthalmology 2013;
131(8):1085.

A 55-year-old woman presented with eyelid swelling associated with irritation, excessive tearing, and
mattering for over 1 month.

A B C

Printed with permission from Lee V., Azari A.A., Nehls S., Potter H.D., Albert D.A. Leiomyoma of the Lower Eyelid. JAMA Ophthalmology 2013;
131(8):1085.

245
ANSWER â•… Case 123

Diagnosis:╇ Leiomyoma of the conjunctiva.

Clinical description:╇ Slit-lamp examination reveals an extensive red, elevated conjunctival lesion.

Histological description:╇ (A) Histopathological evaluation demonstrates conjunctival tissue with an area of
smooth muscle bundles. Higher magnification demonstrates fascicles composed of fusiform cells with cigar-
shaped end nuclei (B). Cytoplasmic staining with muscle-specific actin stain is observed (C).

A B C

246
Chapter 3â•… Cornea and conjunctiva CASE 124 â•…

A 63-year-old man presented with a rapidly growing conjunctival mass. The conjunctival mass was
removed and submitted for histopathological evaluation.

247
ANSWER â•… Case 124

Diagnosis: ╇ Conjunctival mucosa-associated lymphoid tissue-type (MALT-type) lymphoma.

Clinical description:╇ Clinical examination reveals a red, elevated conjunctival lesion with classic “salmon
patch” appearance.

Histological description:╇ Histopathological examination reveals proliferation of lymphocytes arranged


in sheets.

248
Chapter 3â•… Cornea and conjunctiva CASE 125 â•…

A 66-year-old woman presented with fullness under her right lower eyelid. She later underwent an
excisional conjunctival biopsy.

249
ANSWER â•… Case 125

Diagnosis:╇ Burkitt’s lymphoma.

Clinical description:╇ Clinical examination reveals a “salmon-patch” conjunctival lesion in the left lower
fornix in this patient.

Histological description:╇ Histopathology reveals tissue infiltrated with sheets of lymphocytes with presence
of many tingible body macrophages. Tingible body macrophages are demonstrated with arrows. Note the
“starry sky” appearance. Immunohistochemical staining confirmed the diagnosis of Burkitt’s lymphoma.

250
Chapter 3â•… Cornea and conjunctiva CASE 126 â•…

Courtesy of Sarah M. Nehls, M.D. University of Wisconsin-Madison.

A 42-year-old woman presented with foreign body sensation and a conjunctival mass for the past
6 months.

251
ANSWER â•… Case 126

Diagnosis:╇ Benign reactive lymphoid hyperplasia (BRLH).

Clinical description:╇ Slit-limp examination shows a multilobulated red elevated conjunctival lesion.

Histological description:╇ Histopathology demonstrates tissue with a nonkeratinized stratified squamous


surface epithelium overlying multiple lymphoid follicles (asterisks). The follicles are composed of a germinal
center and a mantle zone surrounding them. The follicular centers contain cleaved and noncleaved medium
sized lymphoid cells with mitotic figures and tingible body macrophages (arrowheads).

Conjunctival
epithelium
*
*

* * *

Germinal center

Mantle zone

252
Chapter 3â•… Cornea and conjunctiva CASE 127 â•…

A 26-year-old woman presented with ocular irritation and foreign body sensation. She underwent
excisional biopsy of the conjunctival lesion shown above.

253
ANSWER â•… Case 127

Diagnosis:╇ Conjunctival inclusion cyst.

Clinical description:╇ An elevated, well-circumscribed, fluid-filled conjunctival mass is seen.

Histological description:╇ A large cystic space lined by nonkeratinized stratified squamous epithelium is seen
within the conjunctival stroma. The cystic space is filled with exudative material.

Conjunctival epithelium

Large cystic cavity filled


with exudates

Nonkeratinized stratified
squamous epithelium
lining the cyst

254
Chapter 3â•… Cornea and conjunctiva CASE 128 â•…

A 43-year-old woman presented with ocular irritation for the past 4 months. She later underwent an
excisional biopsy of the conjunctival lesion.

255
ANSWER â•… Case 128

Diagnosis:╇ Conjunctival lymphangiectasia.

Clinical description:╇ Clinical examination demonstrates thin-walled translucent cystic spaces on the
temporal aspect of the bulbar conjunctiva.

Histological description:╇ Histopathological examination reveals conjunctiva with multi-loculated cystic


spaces (asterisks) in the substantia propria (stroma) lined by thin walls and flattened endothelial cells.

Conjunctival epithelium

*
* *
Conjunctival stroma
*
*

256
Chapter 3â•… Cornea and conjunctiva CASE 129 â•…

From Albert, Daniel M., Miller, Joan W., Azar, Dimitri T., and Blodi, Barbara A. (eds). 2008. Albert
& Jakobiec’s Principles and Practice of Ophthalmology, 3rd ed. Philadelphia: Copyright Elsevier 2008.

A 43-year-old woman presented with a mass under her right upper eyelid. The mass was excised and
evaluated histopathologically.

257
ANSWER â•… Case 129

Diagnosis:╇ Lipodermoid.

Clinical description:╇ Slit-lamp examination reveals a fatty, elevated, supratemporal subconjunctival lesion.
Also noted is the rigid gas-permeable contact lens worn by the patient.

Histological description:╇ Histopathology reveals tissue covered by stratified squamous epithelium. The
stroma is collagenous and contains aggregates of lipid cells.

Surface epithelium

Lipid aggregates

258
Chapter 3â•… Cornea and conjunctiva CASE 130 â•…

A 22-year-old man with a past ocular history significant for conjunctival laceration presented with a
red exuberant conjunctival mass.

259
ANSWER â•… Case 130

Diagnosis:╇ Pyogenic granuloma.

Clinical description:╇ A well-circumscribed, red, elevated lesion with a smooth surface arising from the bulbar
conjunctiva.

Histological description:╇ The histopathology demonstrates an exuberant tissue with an edematous stroma.
Acute and chronic nongranulomatous inflammation and radiating blood vessels are present.

Edematous stroma

Radiating blood vessels

260
Chapter 3â•… Cornea and conjunctiva CASE 131 â•…

From Albert, Daniel M., Miller, Joan W., Azar, Dimitri T., and Blodi, Barbara A. (eds). 2008. Albert
& Jakobiec’s Principles and Practice of Ophthalmology, 3rd ed. Philadelphia: Copyright Elsevier 2008.

A 23-year-old man presented with discharge from his left eye for the past 3 days.

261
ANSWER â•… Case 131

Diagnosis:╇ Bacterial conjunctivitis.

Clinical description:╇ Clinical examination demonstrates conjunctival redness, hyperemia, and mild
discharge.

Histological description:╇ Histological examination reveals tissue lined by goblet-cell-containing,


nonkeratinized epithelium. The conjunctival stroma is infiltrated with lymphocytes and plasma cells (circle).

Goblet cell in
conjunctival
epithelium

262
Chapter 3â•… Cornea and conjunctiva CASE 132 â•…

From Albert, Daniel M., Miller, Joan W., Azar, Dimitri T., and Blodi, Barbara A. (eds). 2008. Albert
& Jakobiec’s Principles and Practice of Ophthalmology, 3rd ed. Philadelphia: Copyright Elsevier 2008.

A 46-year-old patient presented with ocular pain and redness.

263
ANSWER â•… Case 132

Diagnosis:╇ Onchocerciasis.

Clinical description:╇ Slit-lamp examination reveals marked conjunctival redness and chemosis.

Histological description:╇ Histopathological examination reveals a cross-sectional view of the Onchocerca


volvulus.

264
Chapter 3â•… Cornea and conjunctiva CASE 133 â•…

A 10-year-old healthy boy presented with ocular irritation and a large eyelid lesion.

265
ANSWER â•… Case 133

Diagnosis:╇ Ligneous conjunctivitis.

Clinical description:╇ Slit-lamp examination shows a solid “woody” lesion in the superior tarsal conjunctiva.

Histological description:╇ Histopathology demonstrates amorphous eosinophilic substance within the


conjunctival stroma. The conjunctival surface epithelium is not present in this section.

Amorphous eosinophilic
substance

266
Chapter 3â•… Cornea and conjunctiva CASE 134 â•…

A 32-year-old African American woman with a history of shortness of breath presented with a
conjunctival lesion, as shown above.

267
ANSWER â•… Case 134

Diagnosis:╇ Conjunctival sarcoidosis.

Clinical description:╇ Slit-lamp examination reveals a red, elevated subconjunctival lesion in the nasal bulbar
conjunctiva of the right eye.

Histological description:╇ Histopathology demonstrates diffuse infiltration of the conjunctival stroma by


sheets of mononuclear inflammatory cells and tubercles of epithelioid and giant cells. No significant caseation
is present.

Conjunctival epithelium

Lymphoplasmocytic
inflammation

Granuloma

Granuloma
Granuloma

Granuloma

Granuloma

268
Chapter 3â•… Cornea and conjunctiva CASE 135 â•…

Courtesy of Heather D. Potter, M.D. University of Wisconsin-Madison.

The patient is a 57-year-old woman with a history of eye rubbing. She underwent an excisional biopsy
of the left tarsal conjunctiva.

269
ANSWER â•… Case 135

Diagnosis:╇ Lichen simplex chronicus.

Clinical description:╇ Clinical examination after the eversion of the left upper eyelid demonstrates thickening
and laceration of the tarsal conjunctiva.

Histological description:╇ Histopathology demonstrates tissue lined by metaplastic conjunctiva showing


squamatization composed of uniform keratinocytes with compact ortho- and parakeratosis. A mild
lymphocytic infiltrate is seen in the substantia propria. There is loss of collagen density in the tarsus. Glandular
structures are also noted in the substantia propria (asterisk).

Keratinized conjunctival
epithelium with parakeratosis

Acanthosis

Dermis with mild inflammation

270
Chapter 3â•… Cornea and conjunctiva CASE 136 â•…

Courtesy of Neal P. Barney, M.D. University of Wisconsin-Madison.

A 65-year-old woman with chronic ocular discomfort and decreased vision underwent a conjunctival
biopsy.

271
ANSWER â•… Case 136

Diagnosis:╇ Ocular cicatricial pemphigoid.

Clinical description:╇ The clinical image demonstrates corneal opacity, trichiasis, madarosis, and
symblepharon formation.

Histological description:╇ Histopathology demonstrates marked conjunctival inflammation (top).


Immunohistochemistry demonstrates a linear band of IgG in the conjunctival epithelial basement membrane.

Conjunctival epithelium

Conjunctival stroma

Nongranulomatous
conjunctival inflammation

Linear lgG band

272
Chapter 3â•… Cornea and conjunctiva CASE 137 â•…

Courtesy of Neal P. Barney, M.D. University of Wisconsin-Madison.

A 54-year-old woman presented with dry eyes and foreign body sensation. The patient had been using
artificial tears with no relief. She underwent excision of the superior bulbar conjunctiva.

273
ANSWER â•… Case 137

Diagnosis:╇ Superior limbic keratoconjunctivitis.

Clinical description:╇ Marked vascular dilation, hyperemia, and thickening is seen in the superior bulbar
conjunctiva.

Histological description:╇ The specimen consists of conjunctival tissue showing areas of epithelial thickening,
degeneration, and complete loss of goblet cells. Nuclear pseudoinclusions and abnormal aggregations of
chromatin are seen within many cells. A nongranulomatous mixed inflammatory infiltrate is seen within the
substantia propria and epithelium.

Epithelial thickening

Inflammation within
the substantia propria

274
Chapter 3â•… Cornea and conjunctiva CASE 138 â•…

From Albert, Daniel M., Miller, Joan W., Azar, Dimitri T., and Blodi, Barbara A. (eds). 2008. Albert
& Jakobiec’s Principles and Practice of Ophthalmology, 3rd ed. Philadelphia: Copyright Elsevier 2008.

A 6-year-old boy presented with bilateral itching of the eyes and foreign body sensation. A few similar
episodes in the past were treated with topical steroids.

275
ANSWER â•… Case 138

Diagnosis:╇ Vernal keratoconjunctivitis.

Clinical description:╇ The clinical examination demonstrates presence of giant papillae on the tarsal
conjunctiva when the upper eyelid is everted.

Histological description:╇ Histopathological examination reveals conjunctiva with stratified squamous


epithelium. The epithelium shows focal areas with lymphocytic and eosinophilic infiltration. Marked diffuse
eosinophilic infiltration (some are shown with arrows) as well as chronic lymphoplasmacytic inflammation is
seen within a vascular (asterisks) stroma.

Conjunctival epithelium

*
*
*

* * *

276
Chapter 4â•… Uveal tract tumors CASE 139 â•…

A 48-year-old woman presented with decreased vision over the past 8 months. The fundus image and
ultrasound findings are depicted above.

277
ANSWER â•… Case 139

Diagnosis:╇ Choroidal hemangioma.

Clinical description:╇ An elevated subretinal lesion demonstrating high internal reflectivity on A-scan is seen.

Histological description:╇ Histopathology reveals dilated vascular channels that are lined by a single layer of
flat endothelium. The vascular channels are filled with red blood cells.

Dilated vascular channels

Endothelial lining of
blood vessels

278
Chapter 4â•… Uveal tract tumors CASE 140 â•…

A 28-year-old woman presented with a pigmented choroidal mass.

279
ANSWER â•… Case 140

Diagnosis:╇ Choroidal nevus.

Clinical description:╇ Dilated fundus examination demonstrates a darkly pigmented choroidal mass.

Histological description:╇ Proliferation of melanocytic cells is seen within the choroid. The cells appear
benign and mitotic figures are not present.

Pigmented cells within


the choroid

RPE cells

Proliferation of
melanocytes is seen
within the choroid

Sclera

280
Chapter 4â•… Uveal tract tumors CASE 141 â•…

A 47-year-old woman presented with multiple iris lesions. No iris lesions were present in her
other eye.

281
ANSWER â•… Case 141

Diagnosis:╇ Iris nevi.

Clinical description:╇ Elevated, nodular pigmented lesions are seen.

Histological description:╇ Multiple areas of melanocytic proliferation consistent with nevi are seen on the iris
surface (arrows).

Iris sphincter muscle

Posterior pigmented
iris epithelium

Anterior lens
capsule

282
Chapter 4â•… Uveal tract tumors CASE 142 â•…

A 70-year-old woman under treatment for ovarian carcinoma presented with bilateral decreased
vision for the past 2 months.

283
ANSWER â•… Case 142

Diagnosis:╇ Bilateral diffuse uveal melanocytic proliferation (BDUMP), a paraneoplastic syndrome.

Clinical description:╇ The fundus examination reveals bilateral patchy pigmented subretinal lesions.

Histological description:╇ Histopathological examination of collette following enucleation at autopsy reveals


diffuse thickening of the choroid and increase in uveal melanocytic cells.

Diffuse
thickening of
the choroid
is noted

Increase in uveal
melanocytic cells
is seen

284
Chapter 4â•… Uveal tract tumors CASE 143 â•…

A 67-year-old man presented with a new iris lesion, as shown above.

285
ANSWER â•… Case 143

Diagnosis:╇ Iris cyst.

Clinical description:╇ Ultrasound demonstrates a posterior cystic iris lesion.

Histological description:╇ Histopathology reveals a large cystic cavity within the posterior pigmented iris
epithelium.

Cornea

Iris

Sclera
Cyst

Ciliary body Posterior pigmented iris


epithelium

286
Chapter 4â•… Uveal tract tumors CASE 144 â•…

An 8-year-old boy presented with the above eyelid deformity.

287
ANSWER â•… Case 144

Diagnosis:╇ Lisch nodule in neurofibromatosis type 1.

Clinical description:╇ Clinical examination reveals an “S-shaped” eyelid deformity.

Histological description:╇ Nodular aggregates of melanocytes, consistent with a Lisch nodule, are seen in the
anterior iris.

Cornea
Iris

Lisch nodule

Lens

Ciliary body

288
Chapter 4â•… Uveal tract tumors CASE 145 â•…

From Albert, Daniel M., Miller, Joan W., Azar, Dimitri T., and Blodi, Barbara A. (eds). 2008. Albert
& Jakobiec’s Principles and Practice of Ophthalmology, 3rd ed. Philadelphia: Copyright Elsevier 2008.

A 5-year-old boy presented with an iris lesion, present for the past 8 months.

289
ANSWER â•… Case 145

Diagnosis:╇ Juvenile xanthogranuloma presenting with an iris mass.

Clinical description:╇ Slit-lamp examination demonstrates a localized iris plaque.

Histological description:╇ Histopathology reveals tissue infiltrated with inflammatory cells and Touton
multinucleated giant cells (arrows).

290
Chapter 4â•… Uveal tract tumors CASE 146 â•…

A 66-year-old man with insulin dependent diabetes mellitus presented with blurry vision.

291
ANSWER â•… Case 146

Diagnosis:╇ Lacy iris vacuolization in a patient with diabetes mellitus.

Clinical description:╇ The dilated fundus examination shows numerous retinal hemorrhages as well as
neovascularization at the head of the optic nerve.

Histological description:╇ Histopathology reveals lacy vacuolization (arrows) of the posterior pigmented iris
epithelium.

Iris

Lens

292
Chapter 4â•… Uveal tract tumors CASE 147 â•…

A 59-year-old man presented with gradual loss of vision. He later underwent enucleation.

293
ANSWER â•… Case 147

Diagnosis:╇ Choroidal melanoma (epithelioid type).

Clinical description:╇ A large elevated choroidal lesion is seen on dilated fundus examination.

Histological description:╇ A mushroom-shaped mass is seen arising from the choroid. The tumor cells
demonstrate considerable cytoplasm (arrows) with a large nucleus and prominent nucleoli (arrowheads). The
cell boundaries are distinct, and there is lack of cohesion between the tumor cells. An exudative retinal
detachment is also present (asterisk).

Break in the
Bowman’s layer

* Mushroom-shaped
Subretinal
choroidal mass
exudates

294
Chapter 4â•… Uveal tract tumors CASE 148 â•…

A 58-year-old woman presented with loss of vision. The clinical examination findings are depicted
above. She later underwent an enucleation.

295
ANSWER â•… Case 148

Diagnosis:╇ Ciliary body melanoma (epithelioid type).

Clinical description:╇ A large elevated ciliary body mass on dilated fundus examination is seen.

Histological description:╇ Histopathology demonstrates a large mass proliferating from the ciliary body
(top). Large epithelioid cells with abundant cytoplasm, large nuclei, and distinct nucleoli are seen. The cell
borders are distinct and the cells are discohesive.

Epithelioid cell with


large cytoplasm

Large nucleus with


prominent nucleoli

296
Chapter 4â•… Uveal tract tumors CASE 149 â•…

Courtesy of Justin Gottlieb, MD, University of Wisconsin-Madison.

A 37-year-old woman presented with the above incidental finding.

297
ANSWER â•… Case 149

Diagnosis:╇ Spindle cell melanoma.

Clinical description:╇ Dilated fundus examination shows a large elevated pigmented choroidal mass in the
posterior pole.

Histological description:╇ Proliferation of spindle cells with elongated nucleus and inconspicuous nucleoli.
Cell boundaries are indistinct with a syncytial pattern.

298
Chapter 4â•… Uveal tract tumors CASE 150 â•…

A 58-year-old woman presented with a pigmented iris lesion in her left eye.

299
ANSWER â•… Case 150

Diagnosis:╇ Iris melanoma.

Clinical description:╇ A slightly elevated pigmented iris lesion is noted on the slit-lamp examination.

Histological description:╇ Histopathology reveals infiltration of the iris tissue with atypical melanocytes. The
cells demonstrate a large nuclear to cytoplasmic ratio.

Posterior pigmented iris


epithelium

300
Chapter 4â•… Uveal tract tumors CASE 151 â•…

Printed with permission from Lewis, David A., Nehls, Sarah, et╯al. Ciliary Body Medulloepithelioma
in a 10-Year-Old Boy. Arch Ophthalmol. 2012 Jul;130(7):881.

A 10-year-old boy presented with decreased vision and an intraocular mass.

301
ANSWER â•… Case 151

Diagnosis:╇ Medulloepithelioma.

Clinical description:╇ A cystic vascular nonpigmented mass is seen in the anterior chamber.

Histological description:╇ Histopathological evaluation demonstrates cystic spaces filled with hyaluronic acid
(asterisk) surrounded by primitive neuroepithelial cells arranged in sheets, cords, tubules, and rosette-like
formations.

Ciliary body mass

Pigmented
epithelium of
the ciliary
body

302
Chapter 4â•… Uveal tract tumors CASE 152 â•…

From Albert, Daniel M., Miller, Joan W., Azar, Dimitri T., and Blodi, Barbara A. (eds). 2008. Albert
& Jakobiec’s Principles and Practice of Ophthalmology, 3rd ed. Philadelphia: Copyright Elsevier 2008.

A 59-year-old woman with a history of choroidal melanoma treated with plaque therapy.
She subsequently developed a blind painful eye with LP vision and the eye was enucleated.

303
ANSWER â•… Case 152

Diagnosis:╇ Choroidal melanoma after plaque therapy.

Clinical description:╇ Dilated fundus examination reveals a pigmented choroidal lesion surrounded by
atrophic changes in the retina and choroid.

Histological description:╇ A pigmented tumor is seen arising from the choroid and involving the retina. The
tumor is highly necrotic and it is composed of epithelioid cells and pigment-laden macrophages. The retina
overlying the tumor is severely atrophic. The sclera overlying the tumor shows areas of inflammation and
fragmentation.

Atrophic retina

Necrotic tumor
Scleral thinning and
fragmentation

Sclera

Scleral inflammation

304
Chapter 4â•… Uveal tract tumors CASE 153 â•…

A 49-year-old man presented with decreased vision. A choroidal mass was noted on dilated fundus
examination. The ultrasound findings are depicted above. He later underwent an enucleation.

305
ANSWER â•… Case 153

Diagnosis:╇ Balloon cell melanoma.

Clinical description:╇ Dilated fundus examination reveals an elevated dome-shaped choroidal mass with an
overlying retinal detachment.

Histological description:╇ Numerous balloon cells (arrows) exhibiting abundant clear cytoplasm are seen.

306
Chapter 4â•… Uveal tract tumors CASE 154 â•…

An 81-year-old man presented complaining of a gradual decrease in his peripheral vision.

307
ANSWER â•… Case 154

Diagnosis:╇ Ring melanoma of the ciliary body (epithelioid type).

Clinical description:╇ Dilated fundus examination reveals a large ciliary body mass.

Histological description:╇ Histopathological examination demonstrates tumor arising from the ciliary body
(top). Large epithelioid cells with abundant cytoplasm, large nuclei, and prominent nucleoli are seen. The cells
have well-defined borders and they are discohesive.

Epithelioid cell with


large cytoplasm

Large nucleus with


prominent nucleoli

308
Chapter 4â•… Uveal tract tumors CASE 155 â•…

The patient is a 54-year-old man with a history of choroidal mass in his left eye. The patient’s past
medical history is significant for renal cell carcinoma.

309
ANSWER â•… Case 155

Diagnosis:╇ Metastatic renal cell carcinoma to choroid.

Clinical description:╇ The dilated fundus examination reveals an elevated choroidal mass superior to the
optic nerve head.

Histological description:╇ Histopathology reveals a tumor that is composed of rounded-to-polygonal-shaped


cells with abundant clear cytoplasm (clear cells) (some of which are shown with arrows) consistent with renal
cell carcinoma. The stroma of the tumor is highly vascular.

310
Chapter 4â•… Uveal tract tumors CASE 156 â•…

A 68-year-old woman with a history of breast cancer presented with loss of vision in her left eye.

311
ANSWER â•… Case 156

Diagnosis:╇ Metastatic breast cancer to choroid.

Clinical description:╇ Dilated fundus examination reveals a large elevated choroidal lesion within the center
of the macula.

Histological description:╇ Histopathology reveals the choroidal tissue infiltrated with epithelial cells, some
exhibiting “Indian-file” arrangement (arrows). These cells are round with eosinophilic cytoplasm. Areas of duct
formation are also seen (asterisks).

Retina

*
*
* *

Sclera *

312
Chapter 5â•…Retina and vitreous CASE 157 â•…

An 82-year-old man presented with decreased vision and the above OCT findings following a
complicated cataract surgery.

313
ANSWER â•… Case 157

Diagnosis:╇ Cystoid macular edema.

Clinical description:╇ Large cystic cavities are seen within the outer plexiform layer.

Histological description:╇ Large amounts of exudates are seen within the outer plexiform layer.

Inner nuclear layer

Outer nuclear layer

e
n th
w ithi layer
es
dat iform
Exu r plex
e
out

Choroid

Sclera

314
Chapter 5â•…Retina and vitreous CASE 158 â•…

A 73-year-old man presented with decreased vision in his right eye. The fundus appearance and OCT
findings are depicted above.

315
ANSWER â•… Case 158

Diagnosis:╇ Epiretinal membrane (macular pucker).

Clinical description:╇ Fundus examination demonstrates macular puckering. OCT shows the well-delineated
epiretinal membrane.

Histological description:╇ Traction from the epiretinal membrane (ERM) has resulted in retinal folds.

Vitreous cavity

Internal limiting
membrane and ERM

Folded retina

Choroid

Sclera

316
Chapter 5â•…Retina and vitreous CASE 159 â•…

A 78-year-old woman presented with decreased vision in her left eye.

317
ANSWER â•… Case 159

Diagnosis:╇ Macular hole.

Clinical description:╇ Dilated fundus examination reveals a round well-delineated macular hole.

Histological description:╇ Histopathology is consistent with a macular hole.

Retina

Retina

Macular hole covered by a thin band of gliotic


tissue

Choroid

Sclera

318
Chapter 5â•…Retina and vitreous CASE 160 â•…

A 72-year-old man presented with loss of vision in his right eye. The ultrasound findings are depicted
above. The eye later underwent an enucleation.

319
ANSWER â•… Case 160

Diagnosis:╇ Funnel-shaped retinal detachment.

Clinical description:╇ Ultrasound reveals a funnel-shaped retinal detachment.

Histological description:╇ A funnel-shaped retinal detachment is evident on histopathology. The retina


remains attached to the optic nerve and ora serrata.

Cornea

Lens

Detached retina

Optic nerve

320
Chapter 5â•…Retina and vitreous CASE 161 â•…

A 71-year-old man with a longstanding history of hypertension presented for a routine eye
examination.

321
ANSWER â•… Case 161

Diagnosis:╇ Cotton-wool spots.

Clinical description:╇ Fundus examination reveals multiple fluffy white patches.

Histological description:╇ Cytoid bodies which are swelled axons are seen (circles). The cytoid bodies form
as a result of nerve fiber layer ischemia and infarction.

Ganglion cell layer

Outer nuclear layer


Inner nuclear layer

Nerve fiber layer


Outer plexiform layer

Inner plexiform layer

Photoreceptors

322
Chapter 5â•…Retina and vitreous CASE 162 â•…

A 67-year-old man with a history of hypertension presented with decreased vision. Dilated fundus
examination reveals the above findings.

323
ANSWER â•… Case 162

Diagnosis:╇ Central retinal vein occlusion.

Clinical description:╇ The fundus image demonstrates swelling of the optic nerve. Scattered hemorrhages in
all retinal layers give it the classic “blood and thunder” appearance.

Histological description:╇ Histopathology shows retinal hemorrhages in all retinal layers (asterisks).

Inner nuclear layer

∗ Nerve fiber layer

∗ ∗

Photoreceptors
Outer nuclear layer

Choroid

Sclera

324
Chapter 5â•…Retina and vitreous CASE 163 â•…

A 77-year-old woman presented with sudden loss of vision in her left eye and the fundus image
shown above. Visual acuity was 20/20 OD and LP OS.

325
ANSWER â•… Case 163

Diagnosis:╇ Central retinal artery occlusion (CRAO).

Clinical description:╇ The fundus examination demonstrates a cherry red spot typical for CRAO.

Histological description:╇ Atrophy and ischemia of the inner retinal layers are most evident, although all
layers are affected.

Severe atrophy of the


inner retinal layers

Relative preservation
of the outer retina

RPE

Choroid

326
Chapter 5â•…Retina and vitreous CASE 164 â•…

Courtesy of William R. Nunery, M.D. University of Louisville, Louisville KY.

A 2-year-old boy with a 6-month history of leukocoria was brought in by his mother.

327
ANSWER â•… Case 164

Diagnosis:╇ Retinoblastoma.

Clinical description:╇ Clinical examination demonstrates marked leukocoria.

Histological description:╇ The slides reveal an enucleated globe with a large basophilic mass arising from the
outer retina causing a complete retinal detachment and subretinal exudation (long arrow). Homer-Wright
rosettes (short arrow), and Flexner-Wintersteiner rosettes (arrowhead) are seen.

Subretinal exudates

328
Chapter 5â•…Retina and vitreous CASE 165 â•…

A 16-year-old boy presented with progressive loss of vision.

329
ANSWER â•… Case 165

Diagnosis:╇ Retinitis pigmentosa.

Clinical description:╇ Fundus images reveal severely attenuated retinal vessel, waxy pallor of the optic disc,
and “bone spicule” pigmentation in the periphery.

Histological description:╇ There is severe and diffuse atrophy of the retinal photoreceptors, retinal pigmented
epithelium, and outer nuclear layer. The retina is focally gliotic and there is a diffuse migration of the retinal
pigmented epithelial cells around the hyalinized retinal vessels (arrowheads).

Attenuated retina

Choroid

330
Chapter 5â•…Retina and vitreous CASE 166 â•…

A 21-year-old woman presented with the above incidental retinal finding.

331
ANSWER â•… Case 166

Diagnosis:╇ Congenital hypertrophy of the retinal pigment epithelium (CHRPE).

Clinical description:╇ Dilated fundus examination reveals multiple areas of grouped pigmented lesions giving
a “bear track” appearance.

Histological description:╇ Histopathology demonstrates hypertrophy and increased pigmentation of the


retinal pigment epithelium.

Hypertrophied retinal
pigment epithelial layer

332
Chapter 5â•…Retina and vitreous CASE 167 â•…

From Albert, Daniel M., Miller, Joan W., Azar, Dimitri T., and Blodi, Barbara A. (eds). 2008. Albert
& Jakobiec’s Principles and Practice of Ophthalmology, 3rd ed. Philadelphia: Copyright Elsevier 2008.

A 58-year-old man with no ocular complaints and normal visual acuity was seen for a routine eye
examination. A slit-lamp image through his dilated pupil is shown.

333
ANSWER â•… Case 167

Diagnosis:╇ Asteroid hyalosis.

Clinical description:╇ The dilated slit-lamp examination reveals small round-to-oval asteroid bodies
suspended in the vitreous.

Histological description:╇ Histopathology demonstrates round, Alcian blue positive spheres (asterisks)
consistent with asteroid bodies within the vitreous. Many of the asteroid bodies are attached to the vitreous.

*
Vitreous strands

*
Vitreous *

334
Chapter 5â•…Retina and vitreous CASE 168 â•…

A 12-year-old boy presented with decreased vision and the above fundus finding.

335
ANSWER â•… Case 168

Diagnosis:╇ Coats’ disease.

Clinical description:╇ Fundus image demonstrates retinal exudates.

Histological description:╇ Histopathology demonstrates retinal detachment and subretinal exudates. Dilated
telangiectatic blood vessels are also present.

Subretinal
exudates

Telangiectatic
blood vessels

336
Chapter 5â•…Retina and vitreous CASE 169 â•…

A 79-year-old man presented for a routine eye examination, with the above retinal finding.

337
ANSWER â•… Case 169

Diagnosis:╇ Senile retinoschisis.

Clinical description:╇ Fundus examination demonstrates smooth elevation of the retina inferiorly.

Histological description:╇ Histopathology demonstrates splitting of the retina at the level of the outer
plexiform layer (OPL).

Inner retina

Splitting at the level of OPL

Outer retina

Sclera

338
Chapter 5â•…Retina and vitreous CASE 170 â•…

From Albert, Daniel M., Miller, Joan W., Azar, Dimitri T., and Blodi, Barbara A. (eds). 2008. Albert
& Jakobiec’s Principles and Practice of Ophthalmology, 3rd ed. Philadelphia: Copyright Elsevier 2008.

A 33-year-old man presented with the above incidental finding on dilated fundus examination.

339
ANSWER â•… Case 170

Diagnosis:╇ Von Hippel–Lindau disease.

Clinical description:╇ The fundus examination reveals dilated retinal vessels feeding a retinal
hemangioblastoma in a patient with Von Hippel–Lindau disease.

Histological description:╇ Histopathology demonstrates dilated vascular channels overlying the optic nerve
(asterisks).

* *

Sclera
Optic nerve

340
Chapter 5â•…Retina and vitreous CASE 171 â•…

From Albert, Daniel M., Miller, Joan W., Azar, Dimitri T., and Blodi,
Barbara A. (eds). 2008. Albert & Jakobiec’s Principles and Practice of
Ophthalmology, 3rd ed. Philadelphia: Copyright Elsevier 2008.

A 71-year-old renal transplant patient presented with decreased vision. Dilated fundus examination
demonstrated the above findings.

341
ANSWER â•… Case 171

Diagnosis:╇ Cryptococcal retinitis.

Clinical description:╇ Dilated fundus examination demonstrates multifocal retinal infiltrates in this renal
transplant patient.

Histological description:╇ Histopathological examination demonstrates many well encapsulated


cryptococcus organisms (arrows).

342
Chapter 5â•…Retina and vitreous CASE 172 â•…

A 72-year-old woman presented with decreased vision and ocular pain. Doppler echocardiography
was suspicious for endocarditis.

343
ANSWER â•… Case 172

Diagnosis:╇ Endogenous endophthalmitis.

Clinical description:╇ Dilated fundus examination shows multiple cream-colored infiltrates within the
vitreous.

Histological description:╇ Histopathology demonstrates an enucleated eye with suprachoroidal hemorrhage,


subretinal hemorrhage, and retinal detachment (top, asterisk). Gram stain demonstrates clusters of Gram-
positive organisms within the vitreous (bottom).

344
Chapter 5â•…Retina and vitreous CASE 173 â•…

A 64-year-old patient presented with decreased vision and eye pain 3 days after a routine cataract
surgery.

345
ANSWER â•… Case 173

Diagnosis:╇ Hypopyon with probable secondary endophthalmitis.

Clinical description:╇ Slit-lamp examination reveals conjunctival hyperemia along with a hypopyon. Radial
corneal scars consistent with previous radial keratotomy are seen.

Histological description:╇ Histopathology demonstrates marked inflammation in the anterior chamber and
the vitreous.

Marked intraocular Cornea


inflammation
Lens fragments

Ciliary body

346
Chapter 5â•…Retina and vitreous CASE 174 â•…

Courtesy of James A. Eadie, M.D. University of Wisconsin-Madison.

A 46-year-old man presented with a 1-week history of decreased vision, photophobia and ocular pain.
On ophthalmic examination, he had variable amounts of vitritis in addition to the retinal findings
shown above.

347
ANSWER â•… Case 174

Diagnosis:╇ Acute retinal necrosis (ARN).

Clinical description:╇ Vitritis is seen, and the retina shows necrosis and opacification.

Histological description:╇ Histopathology demonstrates necrosis and disorganization of all the retinal layers.

Retinal necrosis
and atrophy

Sclera

Necrosis and
atrophy of all
retinal layers

Choroid

Sclera

348
Chapter 5â•…Retina and vitreous CASE 175 â•…

A 43-year-old woman from the Midwest United States presented with decreased vision.

349
ANSWER â•… Case 175

Diagnosis:╇ Ocular histoplasmosis.

Clinical description:╇ A dilated fundus examination reveals multiple punched out chorioretinal lesions in the
periphery and within the macula. Hemorrhages and scarring are also present within the macular region.

Histological description:╇ Histopathological evaluation reveals the Histoplasma capsulation organisms


(arrows).

350
Chapter 5â•…Retina and vitreous CASE 176 â•…

A 22-year-old man presented with a history of decreased vision in his left eye.

Courtesy of Mozhgan R. Kanavi, MD. Ophthalmic Research Center, Shahid Beheshti University of Medical Sciences,
Tehran, Iran.

351
ANSWER â•… Case 176

Diagnosis:╇ Toxoplasma chorioretinitis.

Clinical description:╇ Dilated fundus examination demonstrates extensive chorioretinal scarring in the
macular region in this patient.

Histological description:╇ Histopathology reveals disorganized retinal tissue with the presence of occasional
toxoplasma cysts (arrows).

Courtesy of Mozhgan R. Kanavi, MD. Ophthalmic Research Center, Shahid Beheshti University of Medical Sciences,
Tehran, Iran.

352
Chapter 5â•…Retina and vitreous CASE 177 â•…

A 58-year-old patient presents with decreased vision, photophobia, and pain in his right eye.
The patient had a history of a left ruptured globe 9 years ago. The fluorescein angiogram of the
right eye is shown above.

353
ANSWER â•… Case 177

Diagnosis:╇ Sympathetic ophthalmia.

Clinical description:╇ Fluorescein angiogram demonstrates multiple areas of leakage consistent with serous
retinal detachment.

Histological description:╇ Histopathological examination reveals marked granulomatous inflammation,


consisting of histiocytes and multinucleated giant cells, within the choroid. The choriocapillaris is relatively
spared.

RPE

Histiocytes

Multinucleated giant cells

Sclera

354
Chapter 5â•…Retina and vitreous CASE 178 â•…

From Albert, Daniel M., Miller, Joan W., Azar, Dimitri T., and Blodi, Barbara A. (eds). 2008. Albert
& Jakobiec’s Principles and Practice of Ophthalmology, 3rd ed. Philadelphia: Copyright Elsevier 2008.

A 77-year-old patient with a history of chronic leukemia presented with decreased vision.

355
ANSWER â•… Case 178

Diagnosis:╇ Leukemia.

Clinical description:╇ Dilated fundus examination reveals vitreous haze and multiple nodular choroidal
infiltrates.

Histological description:╇ Histopathology demonstrates exudative retinal detachment. Infiltration of the


choroid and subretinal space with many leukemic cells is seen.

Leukemic cells infiltrating the


subretinal space

Retina Subretinal
exudates
Choroid

Sclera

356
Chapter 5â•…Retina and vitreous CASE 179 â•…

An 85-year-old woman presented for a routine eye examination. The appearance of the fundus is
shown above. Fundus examination of her right eye revealed similar findings.

357
ANSWER â•… Case 179

Diagnosis:╇ Drusen.

Clinical description:╇ The fundus examination reveals numerous yellow deposits within the macula
consistent with drusen.

Histological description:╇ Histopathology demonstrates multiple PAS-positive sub-RPE material consistent


with drusen (asterisks).

∗ Neurosensory retina
id
oro
Ch

Drusen
RPE

Sclera ∗

358
Chapter 6â•… Glaucoma and other disorders CASE 180 â•…

From Kanski JJ. Signs in Ophthalmology: Causes and Differential


Diagnosis. Intraocular Pressure and Angle. Philadelphia: Copyright
Elsevier 2010.

An elderly woman presented with elevated IOP and the above slit-lamp appearance.

359
ANSWER â•… Case 180

Diagnosis:╇ Pseudoexfoliation syndrome.

Clinical description:╇ The slit-lamp examination demonstrates the presence of pseudoexfoliative material on
the anterior lens capsule.

Histological description:╇ Histopathology demonstrates deposition of amorphous pseudoexfoliative material


on the anterior lens capsule resembling iron filings aligned on a magnet (arrows).

Lens epithelium
ule
Anterior lens caps

Lens

360
Chapter 6â•… Glaucoma and other disorders CASE 181 â•…

From Albert, Daniel M., Miller, Joan W., Azar, Dimitri T., and Blodi, Barbara A. (eds). 2008. Albert
& Jakobiec’s Principles and Practice of Ophthalmology, 3rd ed. Philadelphia: Copyright Elsevier 2008.

A 68-year-old woman presented with decreased vision, photophobia, and eye pain after a complicated
cataract surgery.

361
ANSWER â•… Case 181

Diagnosis:╇ Phacoantigenic uveitis.

Clinical description:╇ Slit-lamp examination demonstrates corneal edema with presence of many keratic
precipitates and anterior chamber haze.

Histological description:╇ Histopathology demonstrates zonular pattern of granulomatous inflammation,


composed of neutrophils, lymphocytes, and plasma cells as well as epithelioid cells surrounding the lens.
Remnants of broken lens capsule are also seen.

Lens capsule
Lens

Neutrophils

Lymphocytes and
plasma cells

Epithelioid cells

362
Chapter 6â•… Glaucoma and other disorders CASE 182 â•…

A 58-year-old man with a past medical history of hypertension and insulin-dependent diabetes
mellitus presented with decreased vision and eye pain. IOP was measured to be 65╯mmHg.

363
ANSWER â•… Case 182

Diagnosis:╇ Neovascular glaucoma.

Clinical description:╇ Clinical examination demonstrates florid iris rubeosis.

Histological description:╇ Histopathology reveals an iris tissue with ectropion uveae. Marked iris
neovascularization (arrowheads) and peripheral anterior synechiae closing the angle are present.

Cornea
Descemet's membrane

Ectropion uveae
Peripheral anterior
synechiae closing the
angle

364
Chapter 6â•… Glaucoma and other disorders CASE 183 â•…

A 16-year-old patient presented with the above gonioscopic findings following a blunt eye trauma.

365
ANSWER â•… Case 183

Diagnosis:╇ Iridodialysis.

Clinical description:╇ Gonioscopic examination reveals separation of the iris from its base.

Histological description:╇ Histopathology demonstrates a detached iris.

Cornea

Detached iris

Sclera

Ciliary body

366
Chapter 6â•… Glaucoma and other disorders CASE 184 â•…

From Albert, Daniel M., Miller, Joan W., Azar, Dimitri T., and Blodi,
Barbara A. (eds). 2008. Albert & Jakobiec’s Principles and Practice of
Ophthalmology, 3rd ed. Philadelphia: Copyright Elsevier 2008.

A 19-year-old patient presented to his ophthalmologist following a blunt ocular trauma.

367
ANSWER â•… Case 184

Diagnosis:╇ Angle recession.

Clinical description:╇ Gonioscopic examination reveals widening of the ciliary body band consistent with
angle recession.

Histological description:╇ Histopathology demonstrates a tear into the face of the ciliary body resulting in a
cleavage between circular and longitudinal ciliary body muscles (arrow).

Sclera

Iris

Ciliary body

368
Chapter 6â•… Glaucoma and other disorders CASE 185 â•…

Courtesy of Neal P. Barney, M.D. University of Wisconsin-Madison.

A 56-year-old man sustained an ocular injury. He later developed an elevated IOP and ocular pain.

369
ANSWER â•… Case 185

Diagnosis:╇ Hemorrhagic glaucoma.

Clinical description:╇ Slit-lamp examination demonstrates hyphema with layering of the blood in the
anterior chamber.

Histological description:╇ Histopathological evaluation shows blood in the anterior chamber and within the
angle (arrows).

Cornea

Anterior chamber

Iris

Posterior chamber

370
Chapter 6â•… Glaucoma and other disorders CASE 186 â•…

A 55-year-old man presented with decreased vision and ocular pain. An intraocular mass was
discovered on examination.

371
ANSWER â•… Case 186

Diagnosis:╇ Melanocytic glaucoma.

Clinical description:╇ Ultrasound reveals a large choroidal lesion most consistent with choroidal melanoma.

Histological description:╇ Histopathology demonstrates numerous pigment-laden macrophages within the


trabecular meshwork and the anterior chamber.

Sclera Cornea

Ciliary body Anterior chamber

Pigment-laden macrophages

372
Chapter 6â•… Glaucoma and other disorders CASE 187 â•…

The patient is a 72-year-old man with gradual loss of vision despite maximum medical and surgical
therapy.

373
ANSWER â•… Case 187

Diagnosis:╇ Optic disc cupping secondary to glaucoma.

Clinical description:╇ The optic nerve head is severely cupped with only a small rim of normal tissue
remaining.

Histological description:╇ An excavated optic nerve head is evident on this histopathologic section (arrows)
as well as atrophy of the optic nerve.

Sclera

374
Chapter 6â•… Glaucoma and other disorders CASE 188 â•…

A 77-year-old man with a history of angle closure glaucoma presented with decreased vision and
restricted visual fields. A few years later he underwent enucleation.

375
ANSWER â•… Case 188

Diagnosis:╇ Schnabel’s cavernous dystrophy.

Clinical description:╇ Dilated fundus examination shows a pale optic nerve with severe excavation and
cupping.

Histological description:╇ Histopathology reveals Schnabel’s cavernous degeneration of the retrolaminar


optic nerve exhibiting spongiform changes (asterisks).

Lamina cribrosa
Subretinal exudates
Subretinal exudates

Sclera
Sclera

*
Optic nerve

* *
* *
*

376
Chapter 6â•… Glaucoma and other disorders CASE 189 â•…

From Albert, Daniel M., Miller, Joan W., Azar, Dimitri T., and Blodi, Barbara A. (eds). 2008. Albert
& Jakobiec’s Principles and Practice of Ophthalmology, 3rd ed. Philadelphia: Copyright Elsevier 2008.

The above eye was removed from a stillborn fetus with multiple abnormalities.

377
ANSWER â•… Case 189

Diagnosis:╇ Synophthalmos (Cyclopia).

Clinical description:╇ The enucleated specimen shows fused globes.

Histological description:╇ Histopathology demonstrates globes fused medially (asterisks) sharing a common
optic nerve.

Cornea Iris Cornea

*
* Lens
Iris
*
*
Vitreous cavity
* Vitreous cavity

Retina Retina

Optic nerve

378
Chapter 6â•… Glaucoma and other disorders CASE 190 â•…

From Albert, Daniel M., Miller, Joan W., Azar, Dimitri From Albert, Daniel M., Miller, Joan W., Azar, Dimitri T., and Blodi, Barbara A. (eds). 2008.
T., and Blodi, Barbara A. (eds). 2008. Albert & Albert & Jakobiec’s Principles and Practice of Ophthalmology, 3rd ed. Philadelphia: Copyright
Jakobiec’s Principles and Practice of Ophthalmology, Elsevier 2008.
3rd ed. Philadelphia: Copyright Elsevier 2008.

A 36-year-old patient with a history of chronic headache presented with decreased vision.

379
ANSWER â•… Case 190

Diagnosis:╇ Vogt–Koyanagi–Harada (VKH) syndrome.

Clinical description:╇ Dilated fundus examination and OCT show multiple serous retinal detachments.

Histological description:╇ Histopathology reveals collection of epithelioid cells under the RPE. The choroidal
tissue is infiltrated with lymphoplasmacytic inflammation. Retinal inflammation is also present but not
shown.

RPE cells

a
om
ranul
Eg
-RP
Sub

n
matio
flam
l in
roida
Cho

380
Chapter 6â•… Glaucoma and other disorders CASE 191 â•…

A 75-year-old woman presented with sudden loss of vision and a right afferent pupillary defect.

381
ANSWER â•… Case 191

Diagnosis:╇ Temporal (giant cell) arteritis.

Clinical description:╇ The fundus examination reveals optic nerve edema.

Histological description:╇ The temporal artery demonstrates thickening of intima, discontinuity in internal
elastic lamina and granulomatous inflammation within the intima and media. Multinucleated giant cells are
also seen.

tima
Thickened in

Nongranulomatous
inflammation within
the adventitia Giant cell

382
Chapter 6â•… Glaucoma and other disorders CASE 192 â•…

A 79-year-old woman presented with hypotony with the above ultrasound finding after a complicated
cataract surgery.

383
ANSWER â•… Case 192

Diagnosis:╇ Suprachoroidal hemorrhage.

Clinical description:╇ Ultrasound demonstrates blood in the suprachoroidal space.

Histological description:╇ Large amounts of hemorrhage are seen in the suprachoroidal space. Total retinal
detachment is present.

384
Chapter 6â•… Glaucoma and other disorders CASE 193 â•…

A 67-year-old woman presented with decreased vision and difficulty driving at night. A slit-lamp view
of the lens is shown above.

385
ANSWER â•… Case 193

Diagnosis:╇ Posterior subcapsular cataract.

Clinical description:╇ Dilated slit-lamp examination demonstrates a posterior subcapsular opacity.

Histological description:╇ Histopathology shows collection of morgagnian globules (asterisks) in the


posterior aspect of the lens (arrow).

Lens

*
*
Posterior lens capsule

386
Chapter 6â•… Glaucoma and other disorders CASE 194 â•…

An 87-year-old man presented with difficulty seeing at both distance and near.

387
ANSWER â•… Case 194

Diagnosis:╇ Hypermature cataract.

Clinical description:╇ Slit-lamp examination shows a white mature cataract in this patient.

Histological description:╇ Histological examination reveals an enlarged lens with morgagnian globules
(asterisks) consistent with cataract.

Iris Anterior lens capsule

Cataractous lens

*
*
* *

388
Chapter 6â•… Glaucoma and other disorders CASE 195 â•…

A 55-year-old man with a history of chronic diarrhea presented with decreased vision for the past
4 months.

389
ANSWER â•… Case 195

Diagnosis:╇ Crohn’s disease.

Clinical description:╇ Exudates forming a macular star are seen on the dilated fundus examination.

Histological description:╇ Intestinal biopsy reveals inflammation within the lamina propria. Peyer’s patches,
which are a normal finding, are also seen.

Inflammation within
lamina propria
Intestinal villi

Peyer’s patches

390
Chapter 6â•… Glaucoma and other disorders CASE 196 â•…

From Albert, Daniel M., Miller, Joan W., Azar, Dimitri T., and Blodi, Barbara A. (eds). 2008. Albert & Jakobiec’s Principles and Practice of
Ophthalmology, 3rd ed. Philadelphia: Copyright Elsevier 2008.

An 8-year-old patient presented with decreased vision and amblyopia.

391
ANSWER â•… Case 196

Diagnosis:╇ Anterior polar cataract.

Clinical description:╇ Slit-lamp examination reveals an anteriorly located lens opacity in the pupillary space.

Histological description:╇ Histopathology reveals calcification and cellular proliferation under the anterior
lens capsule.

Cornea

Anterior lens capsule

Calcified material

Lens

Iris

392
Chapter 6â•… Glaucoma and other disorders CASE 197 â•…

A 66-year-old man with a history of multiple myeloma presented with decreased vision, headaches,
and jaw claudication.

393
ANSWER â•… Case 197

Diagnosis:╇ Amyloid deposits in the temporal artery.

Clinical description:╇ The fundus image shows optic nerve edema with a small peripapillary hemorrhage and
a supratemporal cotton-wool spot. The visual field is significant for a complete superior altitudinal defect.

Histological description:╇ Histopathology demonstrates infiltration of amorphous material within the tunica
intima, tunica media, and tunica adventitia (left). Higher magnification shows giant cells surrounding the
amorphous eosinophilic deposits (center). Amyloid staining with Congo red produces birefringence under
polarized light (right).

394
Chapter 6â•… Glaucoma and other disorders CASE 198 â•…

A 28-year-old man presented with a blind painful left eye.

395
ANSWER â•… Case 198

Diagnosis:╇ Phthisis bulbi.

Clinical description:╇ Clinical examination reveals left enophthalmos and exotropia in this patient.

Histological description:╇ Histopathological examination reveals marked osseous RPE metaplasia producing
a cancellous bone shell in the vitreous cavity. The retina is detached, disorganized, and atrophic. The sclera is
thickened and folded and a mild chronic lymphocytic infiltrate is present.

Cornea Site of a
previous buckle

Osseous
metaplasia

Thickened sclera

396
Chapter 6â•… Glaucoma and other disorders CASE 199 â•…

Courtesy of Neal P. Barney, M.D. University of Wisconsin-Madison.

A 22-year-old man presented with pain and decreased vision after blunt trauma to his eye.

397
ANSWER â•… Case 199

Diagnosis:╇ Hyphema.

Clinical description:╇ Layering of blood is noted within the anterior chamber.

Histological description:╇ Histopathology demonstrates marked hemorrhage in the anterior chamber.

Cornea
Blood in anterior chamber

Lens
Iris

398
Chapter 6â•… Glaucoma and other disorders CASE 200 â•…

From Albert, Daniel M., Miller, Joan W., Azar, Dimitri T., and Blodi,
Barbara A. (eds). 2008. Albert & Jakobiec’s Principles and Practice of
Ophthalmology, 3rd ed. Philadelphia: Copyright Elsevier 2008.

An unconscious 4-month-old infant was brought to ER by her caregiver.

399
ANSWER â•… Case 200

Diagnosis:╇ Abusive head trauma (a.k.a shaken baby syndrome).

Clinical description:╇ Dilated fundus examination reveals massive retinal hemorrhages affecting all layers.

Histological description:╇ Histopathology reveals hemorrhages affecting all retinal layers and the optic nerve
sheath. The retinal hemorrhages extend to the ora serrata (asterisks).

* *

Optic nerve sheath


hemorrhage

Nerve fiber layer

Photoreceptors

400
Index of cases

Chapter 1: Eyelid 40 Face and eyelid cavernous 72 Anterior basement membrane


1 Basal cell carcinoma, 1–2 hemangioma, 79–80 dystrophy, 143–144
2 Actinic keratosis, 3–4 41 Angiolymphoid hyperplasia with 73 Band keratopathy, 145–146
3 Squamous cell papilloma, 5–6 eosinophilia (ALHE), 81–82 74 Cystinosis, 147–148
4 Verruca vulgaris, 7–8 75 Salzmann’s nodular degeneration,
5 Seborrheic keratosis (sessile), 9–10 Chapter 2: Orbit and optic nerve 149–150
6 Xanthelasma, 11–12 42 Pleomorphic adenoma (benign 76 Spheroidal degeneration, 151–152
7 Sebaceous cell adenoma, 13–14 mixed tumor), 83–84 77 Keratoconus, 153–154
8 Inverted follicular keratosis, 15–16 43 Dacryoadenitis, 85–86 78 Corneal Dellen, 155–156
9 Rosacea, 17–18 44 Adenoid cystic carcinoma, 87–88 79 Keratoglobus, 157–158
10 Seborrheic keratosis 45 Hemangiopericytoma, 89–90 80 Fuchs’ endothelial dystrophy,
(papillomatous), 19–20 46 Cavernous hemangioma of the 159–160
11 Keratoacanthoma, 21–22 orbit, 91–92 81 Congenital hereditary endothelial
12 Squamous cell carcinoma in situ, 47 Prolapsed orbital fat, 93–94 dystrophy (CHED), 161–162
23–24 48 Solitary fibrous tumor of the orbit, 82 Posterior polymorphous dystrophy,
13 Invasive squamous cell carcinoma, 95–96 163–164
25–26 49 Rhabdomyosarcoma, 97–98 83 Hassall-Henle bodies, 165–166
14 Morpheaform basal cell carcinoma, 50 Schwannoma, 99–100 84 Wilson’s disease, 167–168
27–28 51 Neurofibroma in a patient with 85 Limbal dermoid, 169–170
15 Sebaceous cell carcinoma with neurofibromatosis, 101–102 86 Descemetocele, 171–172
pagetoid spread, 29–30 52 Fibrous dysplasia, 103–104 87 Acanthamoeba keratitis,
16 Pigmented basal cell carcinoma, 53 Eosinophilic granuloma, 105–106 173–174
31–32 54 Wegener’s granulomatosis, 107–108 88 Fungal keratitis secondary to
17 Merkel cell carcinoma, 33–34 55 Juvenile xanthogranuloma with Aspergillus, 00003#sc0110
18 Eccrine hidrocystoma, 35–36 primary orbital involvement, 89 Herpes simplex virus (HSV)
19 Apocrine hidrocystoma, 37–38 109–110 keratitis, 177–178
20 Epithelial inclusion cyst, 39–40 56 Dermoid cyst, 111–112 90 Microsporidial keratitis,
21 Syringoma, 41–42 57 Canaliculitis secondary to 179–180
22 Infundibular cyst, 43–44 Actinomyces, 113-114 91 Bacterial keratitis due to MRSA
23 Pilomatrixoma, 45–46 58 Orbital infection with (methicillin resistant Staphylococcus
24 Trichoepithelioma, 47–48 mucormycosis, 115–116 aureus), 181–182
25 Trichofolliculoma, 49–50 59 Dacryocystitis, 117–118 92 Rhizopus keratitis, 183–184
26 Trichilemmoma, 51–52 60 Transitional cell carcinoma of the 93 Herpes zoster keratitis, 185–186
27 Chalazion, 53–54 lacrimal sac, 119–120 94 Interstitial keratitis, 187–188
28 Molluscum contagiosum, 55–56 61 Orbital lymphoma (small 95 Corneal squamous cell carcinoma,
29 Intradermal nevus, 57–58 B-lymphocytic lymphoma), 121–122 189–190
30 Compound nevus, 59–60 62 Breast cancer with orbital cancer, 96 Epithelial ingrowth, 191–192
31 Malignant melanoma of the eyelid, 123–124 97 Failed DSEK, 193–194
61–62 63 Papilledema due to idiopathic 98 Intrastromal corneal ring segments
32 Floppy eyelid syndrome, 63–64 intracranial hypertension, 125–126 (INTACS), 195–196
33 Dermatochalasis, 65–66 64 Optic nerve drusen, 127–128 99 Pseudophakic bullous keratopathy,
34 Entropion with normal histology, 65 Optic neuritis, 129–130 197–198
67–68 66 Optic nerve sheath meningioma, 100 Failed corneal graft, 199–200
35 Cavernous hemangioma of the 131–132 101 Previous radial keratotomy,
eyelid, 69–70 67 Optic nerve glioma, 133–134 201–202
36 Eyelid varix, 71–72 102 Corneal blood staining, 203–204
37 Eyelid capillary hemangioma, Chapter 3: Cornea and conjunctiva 103 Corneal conjunctivalization,
73–74 68 Lattice dystrophy, 135–136 205–206
38 Melkersson-Rosenthal syndrome, 69 Granular dystrophy, 137–138 104 Corneal foreign body, 207–208
75–76 70 Avellino corneal dystrophy, 139–141 105 Vertical breaks in the Descemet’s
39 Nevus sebaceous, 77–78 71 Macular dystrophy, 141–142 membrane, 209–210

401
Index of cases

106 Primary acquired melanosis (PAM) Chapter 4: Uveal tract tumors 170 Von Hippel-Lindau disease, 339–340
without atypia, 211–212 139 Choroidal hemangioma, 277–278 171 Cryptococcal retinitis, 341–342
107 Benign acquired melanosis (BAM), 140 Choroidal nevus, 279–280 172 Endogenous endophthalmitis,
213–214 141 Iris nevi, 281–282 343–344
108 Primary acquired melanosis (PAM) 142 Bilateral diffuse uveal melanocytic 173 Hypopyon with probable secondary
with atypia, 215–216 proliferation (BDUMP), 283–284 endophthalmitis, 345–346
109 Conjunctival nevus, 217–218 143 Iris cyst, 285–286 174 Acute retinal necrosis (ARN),
110 Caruncular nevus, 219–220 144 Lisch in neurofibromatosis type 1, 347–348
111 Conjunctival melanoma arising 287–288 175 Ocular histoplasmosis, 349–350
from nevus, 221–222 145 Juvenile xanthogranuloma 176 Toxoplasma chorioretinitis, 351–352
112 Metastatic conjunctival melanoma, presenting with an iris mass, 177 Sympathetic ophthalmia, 353–354
223–224 289–290 178 Leukemia, 355–356
113 Pterygium and nevus occurring in 146 Lacy iris vacuolization in a patient 179 Drusen, 357–358
the same area, 225–226 with diabetes mellitus, 291–292
114 Pterygium, 227–228 147 Choroidal melanoma (epithelioid Chapter 6: Glaucoma and other disorders
115 Pinguecula, 229–230 type), 293–294 180 Pseudoexfoliation syndrome,
116 Oncocytoma, 231–232 148 Ciliary body melanoma (epithelioid 359–360
117 Conjunctival amyloidosis, 233–235 type), 295–296 181 Phacoantigenic uveitis, 361–362
118 Conjunctival squamous papilloma, 149 Spindle cell melanoma, 297–298 182 Neovascular glaucoma, 363–364
235–236 150 Iris melanoma, 299–300 183 Iridodialysis, 365–366
119 Hereditary benign intraepithelial 151 Medulloepithelioma, 301–302 184 Angle recession, 367–368
dyskeratosis (HBID), 237–238 152 Choroidal melanoma after plaque 185 Hemorrhagic glaucoma, 369–370
120 Conjunctival intraepithelial therapy, 303–304 186 Melanocytic glaucoma, 371–372
neoplasia (CIN), 239–240 153 Balloon cell melanoma, 305–306 187 Optic disc cupping secondary to
121 Invasive conjunctival squamous cell 154 Ring melanoma of the ciliary body glaucoma, 373–374
carcinoma, 241–242 (epithelioid type), 307–308 188 Schnabel’s cavernous dystrophy,
122 Conjunctival fibrous histiocytoma, 155 Metastatic renal cell carcinoma to 375–376
243–244 choroid, 309–310 189 Synophthalmos (Cyclopia),
123 Leiomyoma of the conjunctiva, 156 Metastatic breast cancer to choroid, 377–378
245–246 311–312 190 Vogt-Koyanagi-Harada (VKH)
124 Conjunctival mucosa-associated syndrome, 379–380
lymphoid tissue-type (MALT-type) Chapter 5: Retina and vitreous 191 Temporal (giant cell) arteritis,
lymphoma, 247–248 157 Cystoid macular edema, 313–314 381–382
125 Burkitt’s lymphoma, 249–250 158 Epiretinal membrane (macular 192 Suprachoroidal hemorrhage,
126 Benign reactive lymphoid pucker), 315–316 383–384
hyperplasia (BRLH), 251–252 159 Macular hole, 317–318 193 Posterior subcapsular cataract,
127 Conjunctival inclusion cyst, 253–254 160 Funnel-shaped retinal detachment, 385–386
128 Conjunctival lymphangiectasia, 319–320 194 Hypermature cataract, 387–388
255–256 161 Cotton-wool spots, 321–322 195 Crohn’s disease, 389–390
129 Lipodermoid, 257–258 162 Central retinal vein occlusion, 196 Anterior polar cataract, 391–392
130 Pyogenic granuloma, 259–260 323–324 197 Amyloid deposits in temporal artery,
131 Bacterial conjunctivitis, 261–262 163 Central retinal artery occlusion 393–394
132 Onchocerciasis, 263–264 (CRAO), 325–326 198 Phthisis bulbi, 395–396
133 Ligneous conjunctivitis, 265–266 164 Retinoblastoma, 327–328 199 Hyphema, 397–398
134 Conjunctival sarcoidosis, 267–268 165 Retinitis pigmentosa, 329–330 200 Abusive head trauma, 399–400
135 Lichens simplex chronicus, 269–270 166 Congenital hypertrophy of the
136 Ocular cicatricial pemphigoid, retinal pigment epithelium
271–272 (CHRPE), 331–332
137 Superior limbic keratoconjunctivitis, 167 Asteroid hyalosis, 333–334
273–274 168 Coats’ disease, 335–336
138 Vernal keratoconjunctivitis, 275–276 169 Senile retinoschisis, 337–338

402
Index

Page numbers followed by “f” indicate Asteroid bodies, attached to vitreous, 334, Breast cancer, 311
figures. 334f metastatic, to choroid, 311f–312f, 312
Asteroid hyalosis, 333f–334f, 334 with orbital cancer, 123f–124f, 124
A Astigmatism, irregular, 201, 201f Burkitt’s lymphoma, 249f–250f, 250
Atypical basaloid cells, 32
Abusive head trauma, 399f–400f, 400 Avellino corneal dystrophy, 139f–140f, C
Acanthamoeba keratitis, 173f–174f, 174 140
Acanthosis, 4, 16f, 18, 22f, 24f, 52f, 238f, Calcification, in pilomatrixoma, 46f
270f Calcified material, 392f
B
Acanthotic epithelium, 240, 240f Calcified mineralized tissue, Gram-positive
Acanthotic lobules, of epithelial cells, 56f Bacterial conjunctivitis, 261f–262f, 262 filaments in, 114, 114f
Acanthotic skin epithelium, 6f Bacterial keratitis, 181f–182f, 182 Canaliculitis secondary to Actinomyces, 113f,
Acquired immunodeficiency syndrome Balloon cell melanoma, 305f–306f, 114
(AIDS), 179, 179f 306 Capillary hemangioma, of eyelid, 73f–74f,
Actinic keratosis, 3f–4f, 4 Band keratopathy, 145f–146f, 146 74
Actinomyces, canaliculitis secondary to, 113f, Basal cell carcinoma, 1f–2f, 2 Cardiac echo-Doppler, 343
114 morpheaform, 27f–28f, 28 Caruncular lesion, 219, 219f
Acute nongranulomatous inflammation, pigmented, 31f–32f, 32 elevated round, 232
260 Basal epithelial pigmentation, 20f pigmented, 220–221, 221f
Acute retinal necrosis, 347f–348f, 348 Basaloid cells Caruncular nevus, 219f–220f, 220
Adenoid cystic carcinoma, 87f–88f, 88 in adenoid cystic carcinoma, 88f Cataract
Adenoma, pleomorphic, 83f–84f, 84 foamy cells lined by, 30f anterior polar, 391f–392f, 392
Adipose tissue, lobules of, 94, 94f islands of, 2f hypermature, 387f–388f, 388
Afferent pupillary defect, 381 with peripheral palisading of nuclei, posterior subcapsular, 385f–386f, 386
Alcian blue staining, 142 48f Cataract surgery, 191, 191f, 313, 345, 361
ALHE. see Angiolymphoid hyperplasia with Basophilic cells, 46f complications of, 383
eosinophilia (ALHE). “Beaten bronze” appearance, 160 Cataractous lens, 388f
Amblyopia, 391 Benign acquired melanosis (BAM), Cavernous hemangioma
Amorphous eosinophilic substance, 266f 213f–214f, 214 of eyelid, 69f–70f, 70, 79f–80f, 80
Amyloid deposits, in temporal artery, Benign mixed tumor. see Pleomorphic of face, 79f–80f, 80
393f–394f, 394 adenoma. of orbit, 91f–92f, 92
Amyloidosis, conjunctival, 233f–234f, Benign reactive lymphoid hyperplasia CD3 staining, in dacryoadenitis, 86
234 (BRLH), 251f–252f, 252 CD20 staining, in dacryoadenitis, 86
Angiolymphoid hyperplasia with Bilateral diffuse uveal melanocytic Cells, rounded-to-polygonal-shaped, with
eosinophilia (ALHE), 81f–82f, 82 proliferation (BDUMP), 283f–284f, abundant clear cytoplasm, 310f
Angle closure glaucoma, 375 284 Central corneal thinning, 186
Angle recession, 367f–368f, 368 Biopsy Central retinal artery occlusion (CRAO),
Anterior basement membrane dystrophy, excisional, with left upper eyelid 325f–326f, 326
143f–144f, 144 fullness, 83 Central retinal vein occlusion, 323f–324f,
Anterior chamber, 370f, 372f orbital 324
blood in, 398f histiocytic cell tumor in, 110 Chalazion, 53f–54f, 54
Anterior corneal nodules, 150 with increasing proptosis, 91, 91f Chemical burn, 205, 205f
Anterior lens capsule, 360f, 388f, 392f Blood, in anterior chamber, 398f Chorioretinal scarring, 352
Anterior orbitotomy, with left upper eyelid Blood vessel lumen, 76f Chorioretinitis, Toxoplasma, 351f–352f, 352
fullness, 111, 111f Blood vessels Choroidal hemangioma, 277f–278f, 278
Anterior polar cataract, 391f–392f, 392 dilated thick-walled, in cavernous Choroidal inflammation, 380f
Anterior stromal opacities, 138 hemangioma of the orbit, 92 Choroidal mass, 305, 309, 309f
Apical snouts, 38f radiating, 260, 260f Choroidal melanoma
Apocrine hidrocystoma, 37f–38f, 38 Bowman’s layer, 144f, 146, 180f after plaque therapy, 303f–304f, 304
Aspergillus, fungal keratitis secondary to, loss of, 158 epithelioid type, 293f–294f, 294
176 Bowman’s membrane, 150 Choroidal nevus, 279f–280f, 280

403
Index

Choroidal tissue, infiltrated with epithelial Conjunctival intraepithelial neoplasia orbital lymphoma and, 122
cells, exhibiting “Indian-file” (CIN), 239f–240f, 240 right lacrimal sac fossa and, 120
arrangement, 312, 312f Conjunctival melanoma, from nevus, showing enlargement of lacrimal gland,
Chronic lymphoplasmacytic inflammation, 221f–222f, 222 86
276 Conjunctival mucosa-associated lymphoid showing left orbital mass with erosion,
Chronic lytic superior orbital lesion, in tissue-type (MALT-type) lymphoma, 108
eosinophilic granuloma, 106 247f–248f, 248 Cyclopia. see Synophthalmos.
CHRPE. see Congenital hypertrophy of the Conjunctivitis Cystic carcinoma, adenoid, 87f–88f, 88
retinal pigment epithelium bacterial, 261f–262f, 262 Cystic cavity, in dermoid cyst, 112, 112f
(CHRPE). ligneous, 265f–266f, 266 Cystic space, 40f
Ciliary body, 366f, 368f, 372f Connective tissue, with spindle-shaped Cystine crystals, 148
Ciliary body melanoma (epithelioid type), fibrocytes, 96 Cystinosis, 147f–148f, 148
295f–296f, 296 Cornea, 135f, 364f, 370f, 372f, 392f Cystoid macular edema, 313f–314f, 314
CIN (conjunctival intraepithelial central thinning, 186, 186f Cysts
neoplasia), 239f–240f, 240 clouding, 203, 203f in adenoid cystic carcinoma, 88, 88f
Coats’ disease, 335f–336f, 336 deposited, 146 epithelial inclusion, 39f–40f, 40
Collagen fibers, 66f edematous, 190, 197–198, 197f, 200, infundibular, 43f–44f, 44
Compound nevus, 59f–60f, 60 204, 204f keratinized stratified squamous
Cone-shape deformity, of cornea, 154 foreign body, 208, 208f epithelial lining of, 40f
Congenital hereditary endothelial haze, 198, 200, 206 pseudohorn, 10f
dystrophy (CHED), 161f–162f, neovascularization, 187–189, 187f, Cytoid bodies, 322, 322f
162 189f Cytoplasmic antineutrophil cytoplasmic
Congenital hypertrophy of the retinal squamous cell carcinoma, 189f–190f, antibody (cANCA), in Wegener’s
pigment epithelium (CHRPE), 331f 190 granulomatosis, 108
Congo red stain, 136, 136f, 140–141, transplant, 181, 181f, 194, 194f–195f,
233–235, 233f–234f 199, 199f D
Conjunctiva Corneal blood staining, 203f–204f, 204
amyloidosis, 233f–234f, 234 Corneal conjunctivalization, 205f–206f, Dacryoadenitis, 85f–86f, 86
biopsy, 147, 213, 213f, 219, 221f, 271 206 Dacryocystitis, 117f, 118
bulbar, 256, 256f, 260, 268, 273–274 Corneal Dellen, 155f–156f, 156 Dacryocystorhinostomy, in dacryocystitis,
epithelial basement membrane, 272 Corneal dendritic lesions, 178 117f
fibrous histiocytoma, 243f–244f, 244 Corneal epithelium, 144f, 186f, 228f Deformity, eyelid, 101, 101f
inclusion cyst, 253f–254f, 254 Corneal findings, 135, 209, 209f Dermatochalasis, 65f–66f, 66
invasive, squamous cell carcinoma, Corneal foreign body, 207f–208f, 208 Dermis
241f–242f, 242 Corneal graft, failed, 199f–200f, 200 in basal cell carcinoma, 2
laceration, 259, 259f Corneal opacities, 136–138, 136f–137f, in entropion, 68f
leiomyoma of, 245f–246f, 246 188–189, 204 granulomatous inflammation in, 18f
lesion, 211, 211f, 213–215, 213f–215f, bilateral, 147, 147f hyperplastic sebaceous glands, 14
217, 217f, 229, 229f, 237, 237f, multiple scalloped, 164 in infundibular cyst, 44f
239, 239f, 241, 241f, 243, 243f, Corneal scars, 177, 177f, 202, 208f in intradermal nevus, 58f
250, 253, 253f, 255, 255f, 267, Corneal tissue, 174, 174f, 180, 182, 200 malignant squamous cells, 26f
267f Corneal ulcer in Merkel cell carcinoma, 34f
elevated, 246 central, 173, 173f, 175, 175f, 183, sclerosed, 28f
lymphangiectasia, 255f–256f, 256 183f in seborrheic keratosis, sessile, 10f
mass, 247, 247f, 251, 251f, 257, 257f, necrotic, 184 in squamous cell papilloma, 6f
259, 259f non-healing, 185–186, 185f in xanthelasma, 12f
nevus, 217f–218f, 218 Cotton-wool spots, 321f–322f, 322 Dermoid cyst, 111f, 112
pigmentation, 216 CRAO. see Central retinal artery occlusion Descemetocele, 171f–172f, 172
sarcoidosis, 267f–268f, 268 (CRAO). Descemet’s membrane, 156, 162, 168, 178,
squamous papilloma, 235f–236f, 236 Crohn’s disease, 389f–390f, 390 194, 364f
superior bulbar, 212, 212f Cryptococcal retinitis, 341f–342f, 342 fragmented, 178f
superior tarsal, 266 Cryptococcus organisms, encapsulated, 342, vertical breaks in, 209f–210f, 210
tarsal, 270 342f Descemet’s stripping endothelial
tissue, 224 CT scan keratoplasty (DSEK), 193–194,
Conjunctival epithelium, 148f, 212f, 214f, chronic lytic superior orbital lesion and, 193f
222, 226f, 228f, 234f, 252f, 254f, 106 Dilated fundus examination, in
256f, 262f, 268f, 272f, 276f of diplopia, 95, 95f papilledema, 126
Conjunctival hyperemia, 346 of left upper eyelid, 85, 85f Dilated pupil, 333, 333f

404
Index

Diplopia, 95, 95f Epithelial inclusion cyst, 39f–40f, 40 Focal parakeratosis, 18


with breast cancer, 123, 123f Epithelial ingrowth, 191f–192f, 192 Follicular keratosis, inverted, 15f–16f, 16
and exophthalmos of the right eye, 97, Epithelial lining, 38f Foreign body sensation, 143, 143f, 149,
97f Epithelioid cells, 362f 149f, 251, 251f, 253, 273, 273f, 275,
and facial pain, 103, 103f giant, in eosinophilic granuloma, 106, 275f
ocular pain and, 105, 105f 106f Fuchs’ endothelial dystrophy, 159f–160f,
with pain and proptosis, 87, 87f with large cytoplasm, 296, 296f 160, 193, 193f
Double vision Epithelioid histiocytes, 46f Fundus examination, 284, 322, 326, 338,
and left eye proptosis, 89, 89f Epithelium, 138f, 140f, 152f, 158, 196f, 340, 357–358, 382
in orbital lymphoma, 121f, 122 220f see also Dilated fundus examination.
proptosis and, 99, 99f acanthotic skin, 6f Fungal keratitis, 175f–176f, 176
Droopy eyelids, 65–66 corneal, 162f Funnel-shaped retinal detachment,
Drusen, 357f–358f, 358 irregular, 150 319f–320f, 320
optic nerve, 128, 128f Erythematous lesion, 240
Dry eyes, 149, 149f, 155, 155f, 273, 273f Excisional biopsy, 221, 225, 255 G
Dual epithelial layers, pleomorphic conjunctival, 249, 249f, 269
adenoma, 84 with left upper eyelid fullness, 83 Giant cell, 382f
Duct formation, 312, 312f Exophthalmos of right eye, diplopia and, multinucleated, 54f, 178f
Ductules, in pleomorphic adenoma, 84 97, 97f Glandular structures, 270
Dystrophy Exudative retinal detachment, 294, 294f Glaucoma
anterior basement membrane, 144 Eye angle closure, 375
Avellino corneal, 140 dry irritated, 63–64 hemorrhagic, 369f–370f, 370
congenital hereditary endothelial, 162 enucleated, 344 melanocytic, 371f–372f, 372
Fuchs’ endothelial, 160 itching, 275, 275f neovascular, 363f–364f, 364
granular, 138 lesion, 225, 225f, 227, 227f, 265, 265f optic disc cupping secondary to,
lattice, 136f pain, 107, 107f, 207, 207f, 361, 363 373f–374f, 374
macular, 142 redness, 207, 207f, 233, 233f, 263, Glioma, optic nerve, 133f–134f, 134
263f Goblet cells, 148f, 206f, 212f, 220, 220f,
rubbing, 269, 269f 262f, 274
E
tearing, 207, 207f, 235, 235f, 245, Goldenhar syndrome, 169, 169f
Eccrine hidrocystoma, 35f–36f, 36 245f Gomori methenamine silver (GMS) stain,
Ectropion uveae, 364f Eyelid 176
Edema capillary hemangioma, 73f–74f, 74 Gonioscopic examination
cystoid macular, 313f–314f, 314 cavernous hemangioma, 69f–70f, 70 in angle recession, 368
optic nerve, 127, 127f deformity, 101, 101f, 287, 287f in iridodialysis, 366
Edematous cornea, 160 droopy, 65–66 Gram-positive cocci, 182
Endogenous endophthalmitis, 343f–344f, floppy, 63f–64f, 64 Gram-positive filaments, in calcified
344 lesion, 1–2, 1f, 225 mineralized tissue, 114, 114f
Endophthalmitis malignant melanoma of, 61f–62f, 62 Gram-positive organisms, in vitreous, 344,
endogenous, 343f–344f, 344 swelling, 245, 245f 344f
hypopyon with probable secondary, varix, 71f–72f, 72 Granular calcified material, in canaliculitis
345f–346f, 346 secondary to Actinomyces, 114
Endothelial cell layer, 162 Granular dystrophy, 137f–138f, 138,
F
Endothelial lining, 70f, 80f 140–141
Endothelium, 154 Facial asymmetry, in fibrous dysplasia, 104 Granuloma, 268f
intracytoplasmic vacuolization of, 82f Facial pain, diplopia and, 103, 103f eosinophilic, 105f, 106
multilayer, 164 Failed corneal graft, 199f–200f, 200 Granulomatous inflammation, 18f, 54f
Entropion, 67f–68f, 68 Fibrotic dermis, 170f intravascular, 76f
Enucleation Fibrous dysplasia, 103f, 104 Grenz zone, 58f
retinal detachment, 319 Fibrovascular core, 8f Guttae, 160, 166
Schnabel’s cavernous dystrophy, 375 Fibrovascular septae, with prolapsed orbital
Eosinophilic granuloma, 105f, 106 fat, 94, 94f H
Eosinophils, 82f Filamentous fungi, branching, 184, 184f
in eosinophilic granuloma, 106, 106f Flexner-Wintersteiner rosettes, 328, 328f Hair follicle, 18f, 28f
Epiphora, 231, 231f Floppy eyelid syndrome, 63f–64f, 64 Hassall-Henle bodies, 165f–166f, 166
Epiretinal membrane (macular pucker), Florid iris rubeosis, 364 Head trauma, abusive, 399f–400f, 400
315f–316f, 316 Fluorescein angiogram, sympathetic Headaches, 379
Epithelial cells, acanthotic lobules of, 56f ophthalmia, 353 chronic, 125, 125f

405
Index

Hemangioma Interstitial keratitis, 187f–188f, 188 Keratitis


cavernous Intervening fibrous septa, 74f bacterial, due to MRSA (methicillin
of eyelid, 69f–70f, 70, 79f–80f, 80 Intervening septae, 70f resistant Staphylococcus aureus),
of face, 79f–80f, 80 Intestinal biopsy, in Crohn’s disease, 182
of orbit, 91f–92f, 92 390 herpes simplex virus (HSV), 178
choroidal, 277f–278f, 278 Intestinal villi, 390f herpes zoster, 186
Hemangiopericytoma, 89f–90f, 90 Intracellular edema, 152 interstitial, 188, 188f
Hemorrhages Intraconal mass, in T1-weighted magnetic microsporidial, 180
central retinal vein occlusion, 324 resonance imaging, 100 Rhizopus, 184
of optic nerve sheath, 400f Intradermal cystic cavity, 38f Keratoacanthoma, 21f–22f, 22
retinal, 400 Intradermal nevus, 57f–58f, 58 Keratoconjunctivitis
suprachoroidal, 383f–384f, 384 Intraepithelial keratin pearls, 24 superior limbic, 273f–274f, 274
Hemorrhagic glaucoma, 369f–370f, 370 Intraocular mass, 301, 301f, 371 vernal, 275f–276f, 276
Henderson-Patterson corpuscles, 56 Intraocular pressure (IOP), neovascular Keratoconus, 153f–154f, 154, 195–196,
Hereditary benign intraepithelial glaucoma, 363 195f
dyskeratosis (HBID), 237f–238f, 238 Intrastromal corneal ring segments Keratoglobus, 157f–158f, 158
Herpes simplex virus (HSV) keratitis, (INTACS), 195f–196f, 196 Keratopathy
177f–178f, 178 Intravascular granulomatous inflammation, band, 145f–146f, 146
Herpes zoster keratitis, 185f–186f, 186 76f pseudophakic bullous, 197f–198f, 198
Hidrocystoma Invasive squamous cell carcinoma, 25f–26f, Keratoplasty, penetrating, 137, 137f, 139,
apocrine, 37f–38f, 38 26 139f, 147, 159, 159f, 163, 173, 175,
eccrine, 35f–36f, 36 Inverted follicular keratosis, 15f–16f, 16 181, 181f, 197, 201, 201f, 203, 203f,
Histiocytic cell tumor, orbital biopsy and, Iridodialysis, 365f–366f, 366 209, 209f
110 Iris, 370f, 392f Keratosis
Histoplasma capsulation organisms, 350, cyst, 285f–286f, 286 actinic, 3f–4f, 4
350f detached, 366f inverted follicular, 15f–16f, 16
Histoplasmosis, ocular, 349f–350f, 350 lesion, 285, 285f, 289, 289f seborrheic, 9f–10f, 10, 19f–20f, 20
Homer-Wright rosettes, 328, 328f multiple, 281, 281f
Hyalosis, asteroid, 333f–334f, 334 pigmented, 299, 299f L
Hyaluronic acid, cystic spaces filled with, melanoma, 299f–300f, 300
302, 302f neovascularization, 364 Lacrimal gland tissue, in dacryoadenitis, 86,
Hyperemia, conjunctival, 346 Iris dilator muscle, 192f 86f
Hyperkeratosis, 4, 4f, 16f, 24, 238f Iris nevi, 281f–282f, 282 Lacrimal sac, transitional cell carcinoma of,
Hypermature cataract, 387f–388f, 388 Irregular trabeculae, of immature woven 119f, 120
Hyperplastic sebaceous glands, 14 bone, 104, 104f Lacy iris vacuolization, in patient with
Hypertension, 321, 323, 363 diabetes mellitus, 291f–292f, 292
Hyphae, multiple branching nonseptate, Lamina cribrosa, 376f
J
184 in optic nerve drusen, 128, 128f
Hyphema, 397f–398f, 398 Juvenile xanthogranuloma Lanugo hairs, 50
Hypopyon, 175–176 presenting with iris mass, 289f–290f, Lattice dystrophy, 135f–136f, 136, 140–141
with probable secondary 290 Left eye proptosis, 89, 89f
endophthalmitis, 345f–346f, 346 with primary orbital involvement, 109f, Left upper eyelid fullness, 83, 83f
Hypotony, 383 110 with dermoid cyst, 111, 111f
in pleomorphic adenoma, 84
K Leiomyoma, of conjunctiva, 245f–246f,
I
246
Immunohistochemistry, 250, 272 Kayser-Fleischer ring, 168 Lens, 362f, 386f, 392f
Inclusion bodies, 56 Keratin cataractous, 388f
Inclusion cyst, conjunctival, 253f–254f, 254 central mass of, 22f Lens capsule, 362f
Increasing proptosis, 91, 91f in infundibular cyst, 44f anterior, 360f, 388f, 392f
Inflammation surface, 10f posterior, 386f
choroidal, 380f in verruca vulgaris, 8f Lens epithelium, 360f
within lamina propria, 390f Keratinized skin epithelium, 34f, 78f Leukemia, 355f–356f, 356
Inflammatory infiltrate, nongranulomatous Keratinized squamous epithelium, Leukocoria, 327
mixed, 274 acanthotic, 10f Leukoplakia, mild, 230
Infundibular cyst, 43f–44f, 44 Keratinized stratified squamous epithelium, Lichens simplex chronicus, 269f–270f, 270
Insulin-dependent diabetes mellitus, 291, 66f, 76f, 170, 170f Ligneous conjunctivitis, 265f–266f, 266
291f, 363 Keratinized surface epithelium, 28f Limbal dermoid, 169f–170f, 170

406
Index

Lipid-laden histiocytes, 12f ring, of ciliary body (epithelioid type), conjunctival, 217f–218f, 218
Lipid vacuole, 54f 307f–308f, 308 melanoma arising from, 221f–222f,
Lipodermoid, 257f–258f, 258 skin, 223, 223f 222
Lisch in neurofibromatosis type 1, spindle cell, 297f–298f, 298 intradermal, 57f–58f, 58
287f–288f, 288 Melkersson-Rosenthal syndrome, 75f–76f, junctional, 226
Lisch nodule, 288, 288f 76 pterygium and, 225f–226f, 226
Lower eyelid fullness, right upper and, in Merkel cell carcinoma, 33f–34f, 34 sebaceous, 77f–78f, 78
rhabdomyosarcoma, 98 Mesenchymal elements, in pleomorphic Nongranulomatous inflammation, 4f, 10f
Lymphangiectasia, conjunctival, 255f–256f, adenoma, 84 within the adventitia, 382f
256 Metastatic conjunctival melanoma, chronic, 54f
Lymphocytes, 362f 223f–224f, 224 Nonkeratinized pseudostratified columnar
Lymphocytic inflammation without Methicillin resistant Staphylococcus aureus epithelium, 118, 118f
necrosis, in dacryoadenitis, 86, 86f (MRSA), 182, 182f Nuclear pleomorphism, 120, 120f
Lymphoma Microsporidial keratitis, 179f–180f, 180 Nucleoli, 294f
Burkitt’s, 249f–250f, 250 Mild lymphocytic infiltrate, 270, 270f Nucleus, large, with prominent nucleoli,
conjunctival mucosa-associated Mild proptosis, in rhabdomyosarcoma, 98 296, 296f
lymphoid tissue-type (MALT- Molluscum contagiosum, 55f–56f, 56 Nystagmus, 161
type), 247f–248f, 248 Monoclonality, in CD3 and CD20 staining,
orbital, 121f–122f, 122 86 O
Lymphoplasmacytic infiltrate, 206 Morgagnian globules, 386, 388
Lymphoplasmacytic inflammation, 268f Morpheaform basal cell carcinoma, Occlusion, central retinal vein, 323f–324f,
27f–28f, 28 324
Motility, restriction of, in Ocular cicatricial pemphigoid, 271f–272f,
M
rhabdomyosarcoma, 98 272
Macrophages, pigment-laden, 372f Mucopolysaccharide deposits, 142 Ocular discomfort, 231, 231f
Macular dystrophy, 141f–142f, 142 Mucor, nonseptate filamentous fungi, 116, chronic, 271, 271f
Macular edema, cystoid, 313f–314f, 314 116f Ocular histoplasmosis, 349f–350f, 350
Macular hole, 317f–318f, 318 Mucormycosis, orbital infection with, Ocular injury, 369
Macular pucker. see Epiretinal membrane. 115f–116f, 116 Ocular irritation, 255, 255f, 265, 265f
Magnetic resonance imaging (MRI) Muir-Torre syndrome, 14 Ocular pain, 143, 143f, 145, 177f, 263,
adenoid cystic carcinoma, 87f–88f, 88 Multi-loculated cystic spaces, 256, 256f 263f, 343, 347, 369, 371
hemangiopericytoma, 90 Multilocular branching lumen, 36 and diplopia, 105, 105f
optic neuritis, 130 Multinucleated giant cell, 54f Ocular shingles, 185, 185f
Malignant melanoma, of eyelid, 61f–62f, Multiple epithelial-lined cysts, 218, 218f Onchocerca volvulus, 264, 264f
62 Munson’s sign, 154 Onchocerciasis, 263f–264f, 264
“Map” dystrophy, 144 Myoepithelial cells, in pleomorphic Oncocytoma, 231f–232f, 232
Masson trichrome adenoma, 84f Ophthalmia, sympathetic, 353f–354f, 354
in rhabdomyosarcoma, 98 Ophthalmic examination, 153, 153f, 165,
staining, 138, 140–141 165f
N
Medial canthus, multiple cystic lesions in, OPL. see Outer plexiform layer (OPL).
37–38 Necrosis Optic disc cupping, 373f–374f, 374
Medulloepithelioma, 301f–302f, 302 acute retinal, 347f–348f, 348 Optic nerve, 376f
Meibomian glands, within tarsus, 64f in Wegener’s granulomatosis, 108f dilated vascular channels in, 340, 340f
Melanocytic cells Neovascular glaucoma, 363f–364f, 364 retrolaminar, degeneration of, 376
nests of, 220, 220f Neovascularization, of iris, 364 Optic nerve drusen, 128, 128f
proliferation of, 280, 280f Nerve bundle, in adenoid cystic carcinoma, Optic nerve edema, 127, 127f
Melanocytic glaucoma, 371f–372f, 372 88f Optic nerve glioma, 133f–134f, 134
Melanoma Nerve fiber layer, 400f Optic nerve head, 374
balloon cell, 305f–306f, 306 Neuritis, optic, 129f–130f, 130 Optic nerve sheath hemorrhage, 400f
choroidal Neurofibroma, with neurofibromatosis, Optic nerve sheath meningioma, 131f–
after plaque therapy, 303f–304f, 101f–102f, 102 132f, 132
304 Neurofibromatosis, neurofibroma with, Optic neuritis, 129f–130f, 130
epithelioid type, 293f–294f, 294 101f–102f, 102 Ora serrata, retinal hemorrhage to, 400
ciliary body, 295f–296f, 296 Neurotrophic ulcer, 186 Orbital biopsy
conjunctival, from nevus, 221f–222f, Neutrophils, 362f histiocytic cell tumor in, 110
222 Nevus increasing proptosis and, 91, 91f
malignant, of eyelid, 61f–62f, 62 caruncular, 219f–220f, 220 Orbital cancer, breast cancer with,
metastatic conjunctival, 223f–224f, 224 compound, 59f–60f, 60 123f–124f, 124

407
Index

Orbital fat, prolapsed, 93f, 94 Primitive skeletal muscle, in Right upper and lower eyelid fullness, in
Orbital infection with mucormycosis, rhabdomyosarcoma, 98, 98f rhabdomyosarcoma, 98
115f–116f, 116 Progressive proptosis, 133 Ring melanoma, of ciliary body (epithelioid
Orbital lymphoma, 121f–122f, 122 Prolapsed orbital fat, 93f, 94 type), 307f–308f, 308
Orbital mass, 109, 109f Proliferated pericytes, hemangiopericytoma Rosacea, 17f–18f, 18
Osseous metaplasia, 396f and, 90, 90f Rosenthal fibers, 134, 134f
Outer plexiform layer (OPL), of retina, 338, Proliferating capillaries, 74f
338f Proptosis, 87, 87f S
Ovarian carcinoma, 283 and double vision, 99, 99f
increasing, 91, 91f S-100 stain, in schwannoma, 100
left eye, 89, 89f “S-shaped” eyelid deformity, 102
P
mild, in rhabdomyosarcoma, 98 “Salmon patch” conjunctival lesion, 248,
Pagetoid spread, 30, 30f progressive, 133 250
Pain, ocular, 143, 263, 263f Pseudoexfoliation syndrome, 359f–360f, Salzmann’s nodular degeneration,
PAM (Primary acquired melanosis), 212, 360 149f–150f, 150
212f, 216, 216f Pseudohorn cysts, 10f Sarcoidosis, conjunctival, 267f–268f, 268
Papillary proliferation, of transitional cells, Pseudophakic bullous keratopathy, Scars, corneal. see Corneal scars.
120, 120f 197f–198f, 198 Schnabel’s cavernous dystrophy, 375f–376f,
Papilledema, due to idiopathic intracranial Pterygium, 227f–228f, 228 376
hypertension, 125f–126f, 126 and nevus at the same area, 225f–226f, Schwannoma, 99f–100f, 100
Papillomatous acanthosis, 20 226 Sclera, 366f, 368f, 372f, 374f, 376f
Parakeratosis, 4, 4f, 16f, 22, 24, 270, 270f Ptosis, 109, 109f in phthisis bulbi, 396
Paraneoplastic syndrome, 283f–284f, 284 Pyogenic granuloma, 259f–260f, 260 Sclerosed dermis, 28f
PAS stain, 140–141 Sebaceous cell adenoma, 13f–14f, 14
Perineural invasion, in adenoid cystic Sebaceous cell carcinoma, 29f–30f, 30
R
carcinoma, 88, 88f Sebaceous gland, 66f, 78f, 170f, 220f
Peripheral anterior synechiae, 364f Racial melanosis, 213f–214f, 214 hyperplastic, 14
Peripheral palisading, 52f Radial corneal scars, 346 Seborrheic keratosis
of nuclei, 2f Reduplicated epithelial basement papillomatous, 19f–20f, 20
Peyer’s patches, 390f membrane, 144f sessile, 9f–10f, 10
Phacoantigenic uveitis, 361f–362f, 362 Renal cell carcinoma, metastatic, to Senile retinoschisis, 337f–338f, 338
Photophobia, 141, 141f, 177, 199, 199f, choroid, 309f–310f, 310 Severe atypia, 190
347, 353, 361 Restricted ocular motility, 107, 107f Shadow cells, 46f
Phthisis bulbi, 395f–396f, 396 Retina, in phthisis bulbi, 396 Skin epithelium, 12f, 24f
Pigment laden macrophages, 212f Retinal detachment, 145, 145f, 344, 344f acanthotic, 6f
Pigmented basal cell carcinoma, 31f–32f, exudative, 356, 356f keratinized, 34f
32 funnel-shaped, 319f–320f, 320 normal, 22f
Pilar unit(s), 6f, 66f, 68f, 170f, 220f Retinal exudates, 336 Skin melanoma, 223, 223f
Pilomatrixoma, 45f–46f, 46 Retinal hemangioblastoma, 340 Skin tags, 9–10
Pilosebaceous units, 60f, 220, 232, 232f Retinal hemorrhages, to ora serrata, 400 Slit-lamp examination, 136, 140, 142–144,
Pinguecula, 229f–230f, 230 Retinal necrosis, acute, 347f–348f, 348 150, 152, 156, 158, 160, 162–164,
Plasma cells, 82f, 362f Retinal pigment epithelium (RPE), 380f 166–167, 167f, 170, 172, 176–177,
Pleomorphic adenoma, 83f–84f, 84 congenital hypertrophy of, 331f–332f, 180, 182, 184, 188, 190, 194, 196,
Pleomorphic spindle-shaped cells, in 332 198, 220, 224, 232, 234, 236, 246,
rhabdomyosarcoma, 98 Retinal vein occlusion, central, 323f–324f, 252, 258, 264, 266, 268
Pleomorphism cells, 34f 324 in anterior polar cataract, 392
Posterior chamber, 370f Retinitis, cryptococcal, 341f–342f, 342 asteroid hyalosis, 334
Posterior pigmented iris epithelium, 192f Retinitis pigmentosa, 329f–330f, 330 in hemorrhagic glaucoma, 370
Posterior polymorphous dystrophy, Retinoblastoma, 327f–328f, 328 in hypermature cataract, 388
163f–164f, 164 Retinoschisis, senile, 337f–338f, 338 hypopyon with probable secondary
Posterior subcapsular cataract, 385f–386f, Retraction artifact, 32, 32f endophthalmitis, 346
386 Retrolaminar optic nerve, degeneration of, in Merkel cell carcinoma, 34
Previous radial keratotomy, 201f–202f, 202 376 in phacoantigenic uveitis, 362
Primary acquired melanosis (PAM) Rhabdomyosarcoma, 97f, 98 in posterior subcapsular cataract, 386
with atypia, 215f–216f, 216 Rheumatoid arteritis, 171, 171f in pseudoexfoliation syndrome, 360
without atypia, 211f–212f, 212 Rhizopus keratitis, 183f–184f, 184 Slit-lamp image, 333
Primary orbital involvement, juvenile Right eyelid swelling, 115, 115f Small B-lymphocytic lymphoma. see
xanthogranuloma with, 109f, 110 Right lacrimal sac fossa, CT scan and, 120 Orbital lymphoma.

408
Index

Solar elastosis, 4f Superior limbic keratoconjunctivitis, V


Solar elastotic changes, 230, 230f, 240 273f–274f, 274
Solitary fibrous tumor of the orbit, 95f–96f, Superior tarsal conjunctiva, 266 V-shaped deformity, 154
96 Suprachoroidal hemorrhage, 344, Vacuolated histiocytes, 244, 244f
Spheroidal degeneration, 151f–152f, 152 383f–384f, 384 Vascular channels, 70f
Spindle cell melanoma, 297f–298f, 298 Supratemporal subconjunctival lesion, 258 dilated, 278, 278f
Spindle-shaped fibrocytes, connective tissue Surface epithelium, 14f, 26f, 40f, 44f, 48f with “staghorn” configuration, 90,
with, 96 keratinized, 28f 90f
Squamous cell carcinoma of skin, 38f dilated blood-filled, 80f
corneal, 190, 190f Surface keratin, 10f lining of, 72f
invasive, 25f–26f, 26 Swelling Vascular “insect wing-like” growth, 228
conjunctival, 241f–242f, 242 pain and, of left upper eyelid, 85 Vascular orbital lesion, in cavernous
nests of malignant, 242 of right lower eyelid, 119, 119f hemangioma of the orbit, 92
in situ, 23f–24f, 24 “Swiss cheese” pattern, tumor with, 88, 88f Vernal keratoconjunctivitis, 275f–276f, 276
Squamous eddies, 16f Sympathetic ophthalmia, 353f–354f, 354 Verruca vulgaris, 7f–8f, 8
Squamous papilloma, 5f–6f, 6 Synophthalmos, 377f–378f, 378 Vision
conjunctival, 235f–236f, 236 Syringoma, 41f–42f, 42 decreased, 125, 125f, 139, 141, 141f,
“Staghorn” configuration, dilated vascular 145, 151, 151f, 159, 161, 161f,
channels with, 90, 90f 163, 163f, 177, 179, 187, 187f,
T 197, 197f, 199, 199f, 271, 271f,
Stem cell
deficiency, corneal conjunctivalization T1-weighted magnetic resonance imaging 277, 277f, 301, 301f, 305, 313,
secondary to, 205f–206f, 206 in schwannoma, 100 315, 317, 335, 341, 343, 347, 349,
transplant, 205, 205f in solitary fibrous tumor of the orbit, 96 351, 353, 355, 361, 363, 371, 375,
Steroids, 189 “Tadpole configuration,” dermis, 42, 42f 379, 385, 389, 391, 393, 397
Storiform pattern, in solitary fibrous tumor Tarsal conjunctiva, 64 and intraocular mass, 301, 301f
of the orbit, 96 Temporal artery, 382 double, 89, 89f
Stratified squamous epithelium, 148, 258, amyloid deposits in, 393f–394f, 394 gradual decrease of, 307
258f, 276 Temporal (giant cell) arteritis, 381f–382f, loss of, 293, 293f, 295, 295f, 311, 311f,
nonkeratinized, 192, 192f, 206, 214, 382 319, 325, 329, 373
218, 218f, 220, 220f, 228, 232, Touton multinucleated giant cells, 290, mild blurring of, 131
232f, 236, 252, 254, 254f 290f poor, 157, 157f
acanthotic, 236, 236f in histiocytic cell tumor, 110, 110f VKH syndrome. see Vogt-Koyanagi-Harada
Striated muscles, 68f Toxoplasma chorioretinitis, 351f–352f, 352 (VKH) syndrome.
Stroma, 138f, 140f, 144f, 180f, 186f, 196f, Toxoplasma cysts, 352f Vogt-Koyanagi-Harada (VKH) syndrome,
226f Transitional cell carcinoma, of the lacrimal 379f–380f, 380
conjunctival, 148f, 212f, 214f, 230, 230f, sac, 119f, 120 Von Hippel-Lindau disease, 339f–340f, 340
254, 254f, 256f, 262, 266, 268, Trauma, head, 399f–400f, 400 von Kossa stain, 146
272f Trichilemmoma, 51f–52f, 52
corneal, 146f, 148, 178f, 198f Trichoepithelioma, 47f–48f, 48 W
scarred, 210, 210f Trichofolliculoma, 49f–50f, 50
edematous, 260, 260f Tumor cells Wegener’s granulomatosis, 107f, 108
fibrosis, 152f cords of, 28f Wilson’s disease, 167f–168f, 168
iris, 192f cytoplasm of, 294f
loss of, 172 X
opacities, 140 U
solar elastosis, 240 Xanthelasma, 11f–12f, 12
substantia propria, 256, 256f Ultrasound
Subretinal exudates, 376f in melanocytic glaucoma, 372
Subretinal hemorrhage, 344 in suprachoroidal hemorrhage, 384
Substantia propria, 270, 274, 274f Uveitis, phacoantigenic, 361f–362f, 362

409
This page intentionally left blank

You might also like